Jump to content

Wikipedia:Reference desk/Science

From Wikipedia, the free encyclopedia

This is an old revision of this page, as edited by Wrk678 (talk | contribs) at 23:12, 9 November 2012 (→‎Deer: new section). The present address (URL) is a permanent link to this revision, which may differ significantly from the current revision.

Welcome to the science section
of the Wikipedia reference desk.
Select a section:
Want a faster answer?

Main page: Help searching Wikipedia

   

How can I get my question answered?

  • Select the section of the desk that best fits the general topic of your question (see the navigation column to the right).
  • Post your question to only one section, providing a short header that gives the topic of your question.
  • Type '~~~~' (that is, four tilde characters) at the end – this signs and dates your contribution so we know who wrote what and when.
  • Don't post personal contact information – it will be removed. Any answers will be provided here.
  • Please be as specific as possible, and include all relevant context – the usefulness of answers may depend on the context.
  • Note:
    • We don't answer (and may remove) questions that require medical diagnosis or legal advice.
    • We don't answer requests for opinions, predictions or debate.
    • We don't do your homework for you, though we'll help you past the stuck point.
    • We don't conduct original research or provide a free source of ideas, but we'll help you find information you need.



How do I answer a question?

Main page: Wikipedia:Reference desk/Guidelines

  • The best answers address the question directly, and back up facts with wikilinks and links to sources. Do not edit others' comments and do not give any medical or legal advice.
See also:


November 4

Syria Rebels

I came across this YouTube video in which the Syrian rebels successfully attack and capture an army outpost, I am confused how they managed to do this because in similar situations in which the US and its allies encounter almost daily in Afghanistan, insurgents will try to attack army bases and the army will call in attack helicopters, which turn on their infrared cameras and can easily see the insurgents even in the day time and kill them with their 30 mm machine gun, or Rockets, Syria has some fairly advanced attack helicopters from Russia. I don't see why they can't do the same. Here is the video:

http://www.youtube.com/watch?v=IC8ENtjVp3I

--Wrk678 (talk) 00:03, 4 November 2012 (UTC)[reply]

Not really a science question, but there are several possibilities:
1) Attack is too rapid to allow such a response. Air responses take time, and the base must be able to hold out until then.
2) Communications are jammed or cut, so they can't call for air support.
3) Syrian outposts aren't designed to withstand attacks from all sides. Many outposts are designed just to defend against one type of attack. For example, posts on the border may be set up only to defend against cross-border attacks.
4) The Syrian air force is simply overwhelmed, and can't respond as quickly as is needed. Lack of spare parts and maintenance issues from constant use can have such an effect.
5) Multiple coordinated attacks stretch the Syrian air force too thin to respond.
Also note that often those bases are retaken by the Syrian military. In many cases, the rebels don't even attempt to hold them, they just raid them and then scatter. StuRat (talk) 00:29, 4 November 2012 (UTC)[reply]


Well, it looks like the attack lasts several hours, starting during the day and ending at dusk. Syria is a pretty small country so I think they could get their in time. I doubt the rebels have any kind of sophisticated jamming equipment that could jam military radios.--Wrk678 (talk) 09:35, 4 November 2012 (UTC)[reply]

Your premise that attacks on military posts in Afghanistan are always quickly suppressed by air attack may not be the whole truth. See 8 U.S. troops killed in Afghanistan, Militant attacks on two remote outposts also leave as many as half a dozen Afghan troops dead.; "It was precisely the kind of attack the top U.S. commander in Afghanistan is hoping to stave off by recently ordering troops to withdraw from such small outposts...". Also the Battle of Wanat and Siege of Sangin. None of these attacks were ultimately successful, but it was sometimes a "damned near-run thing"; although as yiou say, air power was used in a suppressive role in all cases, it's often hard fighting on the ground that wins the day. The training and equipment of the Syrian Army may well not be up to the standard of professional NATO troops. Alansplodge (talk) 10:07, 4 November 2012 (UTC)[reply]
Doing a quick search I find [1] "In recent days it has appeared that Syrian rebels have acquired heavy weapons that have forced the government's air force to bomb rebel-held zones from higher altitude, leading to claims of more indiscriminate bombing." The video shows a helicopter spiraling down and exploding in a ball of fire. Wnt (talk) 13:32, 4 November 2012 (UTC)[reply]
This outpost was a minor one possibly. I doubt they send even one aircraft to all fighting that is taking place. The Syrian_Air_Force#Aircraft_inventory Syrian Air Force has its limitations. What's the point of risking a crown jewel like an attack helicopter to save just half a dozen of trapped soldiers? Comploose (talk) 19:42, 4 November 2012 (UTC)[reply]


Well in Afghanistan fighting on the ground can stave off the assault but it's usually when the attack helicopters come that the insurgents will retreat. Usually there is a "Alamo" call that is given when the base is starting to become overrun that brings all airborne helicopters to the area.

Even though this was a small army base its capture involved the capture and probably the execution of several dozen or more Syrian army soldiers, so I would think the government would prioritize that and send attack helicopters over there. If you watch the end of the video after they capture the base, They hang around there for at least several hours afterwards and nobody shows up, if, for example, it was a priority issue, they probably would have shown up "late" a hour or so after the attack, which they did not.

The thing is I have watched numerous videos of these Syrian rebel attacks, many of them last for hours or more and it doesn't appear that any Syrian attack helicopters usually show up. This makes me wonder if in fact the attack helicopters the Syrians use do in fact have infrared cameras in them. Does anyone have any information on this? --Wrk678 (talk) 21:28, 4 November 2012 (UTC)[reply]

Attack helicopters are not an "I win" button, same as tanks. A chopper or a tank will rarely be sent to enter an engagement with no support. To be effective there has to be a co-ordinated attack. All it takes is one enemy soldier with a cold war era anti-air or a anti-tank weapon to be a serious threat to several million dollars worth of military hardware. With no ground support or good reconnaissance, sending a lone chopper might just be too risky. As is noted above, at least one chopper has already been lost and if I'm not mistaken there's been a few. Vespine (talk) 22:11, 4 November 2012 (UTC)[reply]


Well, it's pretty standard procedure to send attack helicopters to bases being attacked, regardless of anti-air weapons. They had those in Libya and Afghanistan too.--Wrk678 (talk) 22:55, 4 November 2012 (UTC)[reply]

That's an assumption on your part. I see no reason to believe it's "standard procedure". You don't know all the details, as I said, in the other circumstances, maybe there was ground support or reconnaissance available nearby, maybe the threat of anti-air was lower, maybe the pilots were more highly trained; there's just way too many factors involved to say something like "it's standard procedure". Warfare strategy is a very complex topic, it's not just a list of "standard procedures". Vespine (talk) 01:34, 5 November 2012 (UTC)[reply]


I disagree with that, military tactics are fairly universal. --Wrk678 (talk) 01:37, 5 November 2012 (UTC)[reply]

Well, this very scenario seems to disagree with your evaluation. There was a reason why they didn't send attack choppers, I suggested a few explanations, i'm not sure what other answer you would find more satisfactory. Short of asking the "people in charge", which no doubt would not tell you anyway.. I'm not saying that there aren't universal rules of engagement, I was trying to show that there would be lots of exceptions and finer details which civilains would not be privy to. Vespine (talk) 02:34, 5 November 2012 (UTC)[reply]


Well, I'm starting to suspect their helicopters are not equipped with infrared cameras, in which case they wouldn't be of much use in that scenario. If you read about the http://en.wikipedia.org/wiki/Battle_of_Kamdesh it talks about how the solders basically gave up and air support was the only thing that saved them. --Wrk678 (talk) 05:47, 5 November 2012 (UTC)[reply]

Oxidative stress and immune system

Hi all,

Can someone tell me in which treatise (hopefully up-to-date) can I find something about general relationships between the oxidative stress and the cells of the immune system? Thank you very much!

151.71.165.245 (talk) 10:12, 4 November 2012 (UTC)[reply]

This is a very general question, almost too general to answer. (Do you mean the effect of oxidative stress on invading cells, for example, and what level of review do you want?) Get to know http://www.ncbi.nlm.nih.gov/pubmed for making general queries and narrowing them down; http://scholar.google.com has become a very serious alternative. Leaf through oxidative stress, phagocytes including neutrophil, monocyte, and macrophage, phagocytosis, myeloperoxidase, reactive oxygen species, hydrogen peroxide, hypochlorite... it's no small topic, yet one key idea is very straightforward: the immune system recognizes invaders and puts them in special bins where they are bleached to death (and otherwise digested). Wnt (talk) 13:25, 4 November 2012 (UTC)[reply]


Thank you very much. My problem is exactly that I need a general approach to the topic, while the majority of papers I met on-line (including those found using the external links you seggested) is very specific. For this reason I was looking for a treatise. If you are so kind to seggest me a book about the immune system that covers the subject, it will be great!

Many thanks.

193.205.224.192 (talk) 12:54, 5 November 2012 (UTC)[reply]

Black holes at the speed of light

Hi, I have a sort of simple question. If you were to accelerate into a black hole would you travel at the speed of light as you cross the event horizon?

Or if you accelerated in at an angle, so your horizontal component of velocity would be say 100mph and your vertical component would be the increasing velocity due to the acceleration due to gravity of the black hole, could you even hit the speed of light before you hit the event horizon?

(This is assuming you have infinite fuel and are not crushed by the G-force and gravity) — Preceding unsigned comment added by 31.54.166.196 (talk) 14:51, 4 November 2012 (UTC)[reply]

From special relativity no particle with a rest mass can travel at the speed of light. So in answer to your question no you would not be travelling at the speed of light when you cross the event horizon.Dja1979 (talk) 15:58, 4 November 2012 (UTC)[reply]

What would your speed be as you crossed the event horizon? — Preceding unsigned comment added by 86.146.165.159 (talk) 16:17, 4 November 2012 (UTC)[reply]

For an outside observer your speed would be (surprisingly) equal to zero because of gravitational time dilation. Dauto (talk) 16:30, 4 November 2012 (UTC)[reply]
For this reason, it has been argued that black holes (as seen from outside) can never truly exist because they take forever to form! —Tamfang (talk) 20:12, 4 November 2012 (UTC)[reply]
In fact, any observer crossing the event horizon will travel exactly with the speed of light as measured in his own time. Ruslik_Zero 19:13, 4 November 2012 (UTC)[reply]
Is this so? From special relativity, we know that any thing in the universe, that has a rest mass, cannot travel at the speed of light. Thinking about it more, maybe the event horizon is outside the universe (i.e. it doesn't interact with the rest of the universe, it is inside the black hole), so special relativity doesn't hold.Dja1979 (talk) 20:27, 4 November 2012 (UTC)[reply]

So you would travel at the speed of light then? Also how long would it appear to the outside observer that you freeze for? — Preceding unsigned comment added by 86.146.167.230 (talk) 20:08, 4 November 2012 (UTC)[reply]

If an observer sees you freeze, then you never actually make it over in their reference frame, so the answer is forever. However the observation will be redshifted to extreme levels as well, with lower energy photons, and less of them, so eventually nothing will be observed. Graeme Bartlett (talk) 20:15, 4 November 2012 (UTC)[reply]
This has nothing to do with reference frames. No one outside the black hole will ever see you inside because light from inside can't reach the outside. It doesn't matter what speed they're going. -- BenRG (talk) 17:53, 5 November 2012 (UTC)[reply]

As time speeds up relative to how fast you are travelling, does time stop for the person at the event horizon travelling at the speed of light? — Preceding unsigned comment added by 86.146.167.230 (talk) 20:23, 4 November 2012 (UTC)[reply]

Black hole#Event horizon says
According to his own clock, [an observer falling into a black hole] crosses the event horizon after a finite time, although he is unable to determine exactly when he crosses it, as it is impossible to determine the location of the event horizon from local observations.
But Event horizon#Interacting with an event horizon says
An observer crossing a black hole event horizon can calculate the moment they have crossed it, but will not actually see or feel anything special happen at that moment.
"Unable to determine exactly when he crosses it" versus "can calculate the moment they have crossed it" is contradictory. Does anyone know which is right?
A related question: According to those articles someone inside the event horizon can see someone else who previously entered on the same path. Does the previous entrant appear to accelerate to infinite velocity or decelerate to zero velocity? Duoduoduo (talk) 23:49, 4 November 2012 (UTC)[reply]
The point is that the observer is unable to determine the location of the horizon with a local experiment but can calculate it's location if he has full knowledge of the black hole's mass, location, etc. For instance, we could at this very moment be crossing the horizon of a black hole so large that it's center is outside of the observable universe and therefore unknowable. There is no experimental test that we could use to find out whether that's true or not. BTW the guy falling in the black hole is not moving at the speed of light for any local observer as stated above by some people. That's just wrong. Dauto (talk) 00:01, 5 November 2012 (UTC)[reply]
Let's start with the basics. You have a gravity well, and the speed you gain when falling in is the speed you lose when falling out. If you fall into a black hole from a long way away, just barely miss the event horizon somehow and come back out, you should come to a stop at the same long distance away (ignoring gravity waves and such means by which you might lose energy!). Because only something travelling at nearly the speed of light can do this, the speed you must reach near the event horizon is almost the speed of light - relative to the outside observer. But of course, you're falling, so you're at rest the whole time as you see it. If you sped up on the way in though, that doesn't mean you'd be going faster than the speed of light relative to the outside - you'd just have more kinetic energy which, ultimately, would take you flying away from the hole at high speed when you got out. But since mass can acquire an arbitrarily large amount of kinetic energy by going just a little closer to the speed of light, you'd still be below that speed. Ultimately, the outside can only see you where you'd going slower than light relative to their frame of reference; after that, they see... an event horizon. Which is defined as a spot where the math breaks down - you don't really have infinite mass, they can't see you have infinite mass or that you're going at the speed of light, and they're unlikely to get your account about what you saw, though the whole time... you're still at rest. Wnt (talk) 15:10, 5 November 2012 (UTC)[reply]
The math doesn't break down at the event horizon. You can draw an event horizon anywhere in spacetime:
        _______________
       |         /     |   ^
       | inside /      |   |time
       |       /       |   |
       |      / outside|   |
       |_____/_________|    --------> space
If the diagonal line is moving right at the speed of light, it will "behave" just like a black hole event horizon. The inside region "attracts" objects from the outside: you will fall through unless you accelerate away, and the closer you are the larger the necessary acceleration. Nothing from the inside region can ever reach the outside, even light. If someone crosses from the outside to the inside, anyone who remains outside will never see them cross (they'll never see the light, that is). They will see the moment of crossing redshifted into the indefinite future. These makeshift event horizons even emit particles (Unruh radiation), just like black hole event horizons. You are falling through event horizons all the time. -- BenRG (talk) 17:53, 5 November 2012 (UTC)[reply]
Hey BenRG, I like your diagram. It really brings home to a lay person how trivial the concept of a horizon really is. Dauto (talk) 21:47, 5 November 2012 (UTC)[reply]

So why is the nootropic market kind of small?

Why are students not tested for doping? Is there any genius known for taking nootropics. I know that the mathematician Erdos was known for taking amphetamines, and some artists do not hide their past consume of drugs (although art is an entirely different field, you can fuck up with your brain and still be among the best in the field). Comploose (talk) 20:04, 4 November 2012 (UTC)[reply]

At the moment, there are no prohibitions against taking "cognitive enhancers", even for standardized testing. If there are no prohibitions, there is no testing. If, at some point, the use of these is banned — a big if, as it is a complicated issue of performance, mental health, doctor's recommendations, and so on — then presumably testing will have to become part of any effective enforcement regime. --Mr.98 (talk) 20:47, 4 November 2012 (UTC)[reply]
Mathematicians and artists do not really compete with each other as directly as people do in sports, that's why I think perhaps our society doesn't see cognitive enhancement in as bad a light as physical enhancement. Whatever competition there is amongst artists and scientists, it's not the "ends" of those pursuits. Sure you compete indirectly, but you compete indirectly by being born. In competitive sport on the other hand, winning against your opponents IS why you play the sport in the 1st place. I suppose students at one stage will compete for university places and scholarships, but you don't get professional "test takers" who make a living just by taking tests. Vespine (talk) 22:00, 4 November 2012 (UTC)[reply]
Nootropics are generally debatable. Why would there be a way for a drug to improve on intelligence, that nature doesn't already take advantage of? I suppose that if they increased calorie expenditures or something it would be conceivable ... still, what we see are more often things like modafinil. Tell me ... if much of sleep can be conveniently avoided with a little pill, why does every animal species from fruit flies to man go through the process, despite obvious vulnerability to predation and decreased opportunity for food gathering? Wnt (talk) 22:06, 4 November 2012 (UTC)[reply]
Isn't that appealing to nature? The same argument could be made against the bifocals that I'm wearing right now: "Why would there be a way to compensate for myopia, that nature doesn't already take advantage of?". I concur with your overall stance, I just think a different argument would be more convincing.A8875 (talk) 22:34, 4 November 2012 (UTC)[reply]
When we are awake we are on natural stimulants. The reason why animals sleep is due to competetion. If you have some machine that has to repair itself that is performing at some level, then you can put it on stimulants and let it perform better at the expense of its repair capacity. So you need to balance that by having a downtime for repair. Also, performing repairs in downtime is more efficient. Count Iblis (talk) 23:02, 4 November 2012 (UTC)[reply]
I see this less as appealing to nature as appealing to evolution. Evolution, however, is notably poorly tuned for ages that people rarely reached in primitive times, or for activities like reading that were not important many millennia ago, hence the bifocals. Wnt (talk) 01:27, 5 November 2012 (UTC)[reply]
The analogous argument could be made about studying: "Evolution is poorly tuned for activities like memorizing large amounts of information.".A8875 (talk) 03:21, 5 November 2012 (UTC)[reply]
Well, that's a valid point. The idea of general intelligence is a peculiar one, central to understanding humanity, and utterly mysterious. Why did humans evolve so many different skills that everyone has - why aren't there genotypes, families or even races that don't understand how to work calculus, but can do everything else just fine? But we digress... Wnt (talk) 15:34, 5 November 2012 (UTC)[reply]
(after ec) Add to this that the enhancing effect of drugs, any drugs, is rather dubious. That's no drug that will make you wiser. Some will make you kind of restless, reduce your fatigue, keep you awake and so on. But, in my personal experience, people who use them are the kind who need to spend the night awake before exams or people who have mental problems and need to function normally. These groups are not really skewing things to their benefit. OsmanRF34 (talk) 22:12, 4 November 2012 (UTC)[reply]
Ritalin lengthens attention span, and as any student knows, not being distracted while studying is the most important factor in doing well. Some of that is due to the placebo effect, but there's a statistically significant portion that isn't. Testing students for doping is infeasible because students don't confine their studying to a single 1-minute competition every month or year; they study 6 days a week, for half of their waking lives. --140.180.252.244 (talk) 00:14, 5 November 2012 (UTC)[reply]
  • Come to think of it, a main reason why nootropics are not yet popular is that there has been little effort to wage a "war" on them. (Unless you count methamphetamine, which has much in common with Ritalin, if that counts) I think if we had a major DEA push to put people selling modafinil in jail for twenty years, the merchants with good police connections would make a huge profit, giving them a motive to hype it as a "hip" thing in the media and on the street; before long people would be coming up with super-potent addictive analogs to take by injection and we'd be hearing stories of people going without sleep for six months until their brains turn to tapioca. After that a multibillion-dollar trade in it would never end. Wnt (talk) 01:27, 5 November 2012 (UTC)[reply]
I am not sure why you would conclude they are "not yet popular." Amongst those with money and incentive to do well academically, they are immensely popular. A ridiculous percentage of kids at any high-end private high school has prescriptions to them, and many others purchase them illegally from those who do. They are not recreationally very popular, but in terms of getting ahead at the junctures in life that make quite a lot of difference — standardized testing, getting into a "good" school, etc. — and are actually competitive (which is to say, zero-sum — there are winners and losers), there is a quiet little arms race going on. I do suspect that someday there will be some consensus about some standards here, about whether the ability to pay for concentration tilts the scale too much in favor of those with means, but we aren't there yet, just the early murmurs of ethical concern. The issue is a genuinely complicated one, ethically and medically speaking. --Mr.98 (talk) 13:20, 5 November 2012 (UTC)[reply]
The question is open to discussion from a medical standpoint, but I don't see that it skews the playing field towards privileged people. I don't believe many people are not able to pay for generic Ritalin. Filling a prescription through Walmart costs only $4/month, and maybe the poor at the bottom won't be able to buy this generic Ritalin, but most people would be able to afford it, if they want to.
Ritalin has the same problem as other nootropics. Although they might help you perform better if you are late for preparing yourself for one of the 'junctures', I do not believe it will make you any better long term regarding developing and using certain abilities like writing, knowledge acquisition, social abilities, creativity, planning. I am not trying to say that nature made us perfect the way we are, but Ritalin specifically, which works in a similar way to cocaine and amphetamine, has many drawbacks, and the effects on long term users are not well understood yet. It certainly makes you more awake for a certain time, but I believe this is probably at cost of your memory, health (mental and physical), and a rebound effect. I don't see the advantage here. It looks like a real arms race, when both sides invest lots of resources in things that they need, just because the other side have it. OsmanRF34 (talk) 14:53, 5 November 2012 (UTC)[reply]

Coffee and tobacco? μηδείς (talk) 02:24, 6 November 2012 (UTC)[reply]

engine diagram

Hi : Looking for a engine diagram and info.for a toyota avalon xl 2006 . Thank You — Preceding unsigned comment added by Carol9905 (talkcontribs) 20:28, 4 November 2012 (UTC)[reply]

Your local library may have repair and service manuals for that car. Libraries often buy volumes that cover repair procedures for a manufacturer's entire line over 4 years or so. If it is a vehicle that you plan on self-servicing on a regular basis, you might want to look into buying the book made specifically for that car from an auto parts store. 209.131.76.183 (talk) 13:43, 5 November 2012 (UTC)[reply]
Also, have a look at this Google Image search and see if there's anything helpful there (not much of a mechanic myself). Alansplodge (talk) 16:24, 5 November 2012 (UTC)[reply]

Confusion about how telescopes work

I don't really get how a Newtonian telescope works.

Rays of light reflecting off the object we're attempting to view enter into the telescope tube effectively parallel because of its huge distance from us. The rays then reflect off of a parabolic mirror, which focuses all the rays to a point. At least it would, were it not for another mirror obliquely oriented, which reflects these rays to the telescope's eyepiece, which magnifies the image.

Here's my problem: if all the rays are focused to the same point after reflecting off the parabolic mirror, how can we distinguish between different points on the object? Wouldn't the image of the object just be a point? 65.92.7.202 (talk) 22:25, 4 November 2012 (UTC)[reply]

All parallel rays focus to the same point. Non-parallel rays do not. If you imagine 100 rays travelling perpendicular to the mirror's axis, they focus at a certain point on the focal plane. If you imagine 100 rays travelling at an angle of alpha to the perpendicular, those rays focus to a different point on the focal plane. The distance between these 2 focal points is proportional to alpha. --140.180.252.244 (talk) 00:04, 5 November 2012 (UTC)[reply]
Even your eyeball focuses light into a point, but unless there's something wrong with your eye, the point isn't on your retina. Maybe an image like this will help. I think a similar thing happens in the telescope. Vespine (talk) 05:06, 5 November 2012 (UTC)[reply]
Are you sure you don't have it backward? Unless there's something wrong with your eye, the image is on your retina. If it's not, you either have myopia or hyperopia --140.180.252.244 (talk) 07:52, 5 November 2012 (UTC)[reply]
Yes, the image is on the retina in a normal eye, but Vespine's point is that the focal plane lies between the lens and the retina. This whole thread arises from mistakenly assuming that the focal plane is the image plane. Gandalf61 (talk) 14:55, 5 November 2012 (UTC)[reply]
In a Newtonian telescope the focal plane is the image plane. The first answer by 140.180.252.244 is correct, and Vespine's followup isn't helpful as far as I can tell. -- BenRG (talk) 18:04, 5 November 2012 (UTC)[reply]
Ah, right — in astronomy the image plane "is" the focal plane because the objects viewed are "at infinity", unlike in most daily uses of the eyes. —Tamfang (talk) 19:57, 5 November 2012 (UTC)[reply]
Hang on, the focal point is different then the focal plane. The OP was talking about the focal point. You know, the spot where the light focuses to a point if you shine the sun through a magnifying glass. In a Newtonian telescope, that point doesn't fall on your retina. That's what I was trying to show with the image I linked. Vespine (talk) 21:37, 6 November 2012 (UTC)[reply]

Light slow motion

Is this real http://www.youtube.com/watch?v=SoHeWgLvlXI&feature=related. If it is I want to see the effects of relativity. I'm sick of hearing "oh yea relativity has been experimentally verified countless times" I want to see it with naked eyes. Why didn't they do that? Money is tight (talk) 22:42, 4 November 2012 (UTC)[reply]

I don't think that clip shows anything not explicable by classical electrodynamics; you see a light wave propagating, which is impressive, but not so much related to relativity. The easiest way to observe relativity with your eyes would probably be like they did it in 1919. You'd need to observe the stars around the sun (with a telescope ofc) during a total solar eclipse, and then compare the positions of the same stars with a picture you took at night when there was no eclipse. You should see the effect of the sun bending the light of the stars. - Lindert (talk) 23:13, 4 November 2012 (UTC)[reply]
The most dramatic visual demonstration of relativity is probably the Einstein ring. That article has lots of photos. --140.180.252.244 (talk) 00:00, 5 November 2012 (UTC)[reply]
Einstein's prediction before the eclipse was not that gravity would bend light – that was predicted classically – but how much. An Einstein ring doesn't tell us anything about general relativity unless we know the mass of the galaxy in front — or does it? —Tamfang (talk) 00:10, 5 November 2012 (UTC)[reply]
They can get a source traveling at half the speed of light and emits light together with a source that stays still, I want to see these two light beam travel at the same speed. Money is tight (talk) 05:51, 5 November 2012 (UTC)[reply]
To accelerate a macroscopic object (a light source) to such speeds is impractical if feasible at all with our current technology. And where would you film such an event? The light source would leave earth sooner than you can blink your eyes. - Lindert (talk) 08:59, 5 November 2012 (UTC)[reply]
It takes around 0.155mc² to accelerate an object to half the speed of light (γ=1/√0.75), so around 3.32 kilotons of TNT for a 1g object. Given enough time and monetary resources, getting an undersized LED throwie to 0.5c should be right near the edge of humanity's current technological capabilities, but unless you have Scrooge McDuck-levels of wealth, it's just not going to happen. And even if you do happen to have that kind of wealth, you're going to need to pull some strings with the UN, because I doubt they'll think "because I don't believe in special relativity" is a good enough reason to allow you build what is essentially a weapon of mass destruction.--Link (tcm) 15:22, 5 November 2012 (UTC)[reply]
Assuming your LED can survive 1,000,000 g acceleration, it will take around 15 seconds to accelerate it to 0.5c, during which time it will move about 1.5 million km. This is not within the capabilities of the human race now or in the foreseeable future. -- BenRG (talk) 17:32, 5 November 2012 (UTC)[reply]
Would you be satisfied with the Advanced Light Source, where the light is emitted by charged particles travelling at >99% of c, or the Large Hadron Collider, where collisions happen at such a rate (~1 billion per second) that they have to take careful account of the light-speed delay in order to correlate the results from different parts of the detectors?
Anyway, there's nothing strange about the speed of a wave being independent of its source. The same is true of sound waves, for example. -- BenRG (talk) 17:32, 5 November 2012 (UTC)[reply]

Yes the invariance of speed of light is very strange, because it predicts things like length contraction and time dilation. I just find it very difficult to believe and want to see it for myself. Money is tight (talk) 01:02, 6 November 2012 (UTC)[reply]

Unfortunately the speeds and energies involved are so incredibly high that there aren't many good ways to easily observe the effects directly. I recommend you look through Tests of general relativity and Tests of special relativity, and attempt to understand how some of the experiments worked and why the results confirm relativity. If there are tests with methods that you doubt or ones where you don't understand what the test is supposed to be doing, people here will be more than happy to help you work out the details. Some of the tests are pretty simple, but still very difficult to pull off on your own due to the precision required. 209.131.76.183 (talk) 15:32, 6 November 2012 (UTC)[reply]
The reason it's hard to see evidence of modern physics is that if it were easy it would have been noticed much sooner, and hence wouldn't be modern. -- BenRG (talk) 22:26, 6 November 2012 (UTC)[reply]


November 5

In this article, it states there are 2 different units. I don't understand how those 2 different units can end up expressing the same thing in the end?174.20.101.190 (talk) 01:21, 5 November 2012 (UTC)[reply]

They are equivalent: A8875 (talk) 01:50, 5 November 2012 (UTC)[reply]
And, they help remind physicists about the two commonplace ways we calculate impulse: the impulse of an event is often computed by multiplying the net force by the duration of the event; or, the impulse of an event is calculated by multiplying a mass by its total change in velocity. Nimur (talk) 03:07, 5 November 2012 (UTC)[reply]

Mechanism from DNA to proteins etc?

Is the mechanism between DNA and actual proteins etc that are created mapped out?, such that one can use the DNA to calculate (dry run a cell) how the produced protein etc looks like? Electron9 (talk) 03:58, 5 November 2012 (UTC)[reply]

See Transcription (genetics), Translation (genetics), Genetic code, Protein structure prediction. It's probably best for you to make a quick first pass through these articles and say what parts of the topic still confuse you after that. Wnt (talk) 04:16, 5 November 2012 (UTC)[reply]
Yes, specifically genetic code. The genetic code is the recipe for translating from sequences of DNA bases to the sequences of amino acids that make up proteins. It is astonishingly simple. If you know the code, you don't have to know anything about the mechanism that implements it in order to predict the result. (Well, there are a few complications, but they can be ignored on a first pass.) Looie496 (talk) 05:12, 5 November 2012 (UTC)[reply]
Yes, the genetic code is astonishingly simple. But you also have to take into account post-transcriptional modifications and posttranslational modifications. And when you have done that, you only have a protein's amino acid sequence. Inferring a protein's structure and function from its sequence is very difficult. Then you need to work out how it interacts with other proteins and chemicals in metabolic pathways. Then there are the complications of gene regulation. Taking all that into account, being able to "dry run a cell" (however we interpret that) is going to be a massively complex undertaking. The genetic code is just a snowflake on the tip of the iceberg here. Gandalf61 (talk) 12:17, 5 November 2012 (UTC)[reply]
The idea (dry run) is to make a computer simulation that is feed with the DNA code. Once the proteins etc has left the cell other software can simulate the chemical reactions. As for fenotypic gene enable/disable I guess those are in turn controlled by other genes and their resulting structures interacting with environmental chemistry as a feed back loop. Is that mechanism fully understood? Electron9 (talk) 16:12, 5 November 2012 (UTC)[reply]
No I don't suppose it is "fully understood", or even close to that, except in very limited cases. The cellular model article makes interesting reading. It says "The complex network of biochemical reaction/transport processes and their spatial organization make the development of a predictive model of a living cell a grand challenge for the 21st century.". According to this report, the best we can do so far is to build a computer model of mycoplasma genitalium, a bacterium with just 525 genes. Gandalf61 (talk) 17:11, 5 November 2012 (UTC)[reply]
(e/c) The DNA will tell you the order of amino acids, but knowing the order of the amino acids tells you very little about the functionality, which is often determined by the shape of the resulting protein. There's some active research on exactly how that folding works and how those shapes are determined, including a distributed computing project for the "heavy lifting" of just crunching how all of those pieces fit together. In all likelihood there are important mechanisms we aren't even aware of. 150.148.0.65 (talk) 17:34, 5 November 2012 (UTC)[reply]
Hot off the presses: [2] Wnt (talk) 23:17, 7 November 2012 (UTC)[reply]

Maybe not a science question?

Clinical laboratory science, and maybe more of a legal question than anything else, but every set of clinical laboratory results I've ever seen has a statement that says that partial reproduction of the test report is not permitted. Is this just a "don't interpret single test results without seeing context" warning? 150.148.0.65 (talk) 17:34, 5 November 2012 (UTC)[reply]

Is a full reproduction allowed, and a partial not? Otherwise, if neither a full nor a partial reproduction is allowed without consent, that would be a common copyright notice. OsmanRF34 (talk) 18:02, 5 November 2012 (UTC)[reply]
To quote some sample text "Partial reproduction of this Test Report is not permitted" (at top of each page of a set of test results). The sample is from a Indian reference laboratory, but I've seen a lot of the same in American test results. It says nothing about prohibition of full reproduction. Looking at a couple of other things I have lying around it isn't 100%, one of the ones I found (American) just explicitly labels full and partial reports. 150.148.0.65 (talk) 18:29, 5 November 2012 (UTC)[reply]
I think any claim that partial reproduction is forbidden may be void due the "Quotation#Copyright law". In addition foreign law isn't likely enforceable in another country. Electron9 (talk) 18:59, 6 November 2012 (UTC)[reply]

November 6

Dichromacy and Trichromacy colorblind

I was aware some people are colorblind. I was wondering if there is any difference between dichromacy and trichromacy is any difference. If people are having trouble seeing green are they also unable seeing red. is there any difference between red and green colorblindness. I have never heard about anybody having trouble seeing yellow although Color blindness mentions blue-yellow colorblind they basically having difficulty seeing violet hues, but I never heard anybody who is disrupted on yellow hues. I am guessing most people I was aware of is Anomalous trichromacy, but I was wondering how Anomalous trichromacy works on blue-yellow sensors? Are they totally unable to see blue-yellow sensors on Anomalous trichromacy or they just see those as shades of brown. I have met one people on the bus is totally deficient on red, green, and blue they can see grayscale and shades of brown and pink, what type of colorblind is that?--69.226.43.174 (talk) 00:38, 6 November 2012 (UTC)[reply]

I think you misunderstand the meaning of the word colorblindness. It means an inability to distinguish colors that most people can tell apart, not an inability to see colors that most people can see. Looie496 (talk) 00:56, 6 November 2012 (UTC)[reply]
This page purports to show you how your favorite webpage looks to several common kinds of colorblindness. —Tamfang (talk) 01:03, 6 November 2012 (UTC)[reply]
I got the better understanding now about how colorblind works now. I bookmarked the Colorblind Webpage Filter (All I got to do is to type it on Google), and first I went through my favorite websites to play around with the colors. Anomalous colorblind is the less serious type and it is less noticeable unless under certain lighting conditions where certain cones may be skewed and interrupted. Dichromacy means people can only see two or three general colors, and their perceptions are reduced to whatever sensors are remained in their cones.--69.226.43.174 (talk) 04:45, 6 November 2012 (UTC)[reply]

Reason why we don't fall through the floor

I was having a discussion with someone on why matter doesn't simply pass through other matter (given the fact that normal matter is mostly empty space). I was under the impression that it was due to the electromagnetic force. He said that's a common misconception and it is due to the pauli exclusionary principle. Is he correct? ScienceApe (talk) 00:45, 6 November 2012 (UTC)[reply]

I have looked at Pauli exclusion principle and I am confident it isn't reasonable to apply this principle to explain why matter doesn't simply pass through other matter. Dolphin (t) 00:49, 6 November 2012 (UTC)[reply]
(This is incorrect—see below.) -- BenRG (talk) 23:13, 6 November 2012 (UTC)[reply]
It's because those atoms are bonded together as a solid (if the atoms or molecules aren't bonded together, then you have a fluid). Think of a hammock. It's mostly empty space, but nonetheless stops you from falling on the ground, because the threads are all tied together. StuRat (talk) 01:50, 6 November 2012 (UTC)[reply]
You are correct, he is wrong. The exclusionary principal describes why other electrons can't fill already occupied orbitals, hence limiting atomic bonding to specific valences. Sheesh, the things people come up with! μηδείς (talk) 02:18, 6 November 2012 (UTC)[reply]
(This is incorrect—see below.) -- BenRG (talk) 23:13, 6 November 2012 (UTC)[reply]
The Pauly exclusion principle: "Anyone with taste will exclude themself from any movie staring Pauly Shore." StuRat (talk) 03:26, 6 November 2012 (UTC) [reply]
The last exclusionary principal I encountered was my high school headmaster who kicked me out of the class for ... well, I'd better not say. -- Jack of Oz [Talk] 05:23, 6 November 2012 (UTC) [reply]
Wouldn't you think the "headmaster" would be tolerant of such things ? StuRat (talk) 05:47, 6 November 2012 (UTC) [reply]
Only if Jack was disciplined for being a cunning linguist. --Jayron32 05:51, 6 November 2012 (UTC)[reply]
I think they invented tolerance one Monday in 1983, well after I went to school. -- Jack of Oz [Talk] 06:20, 6 November 2012 (UTC) [reply]

The other person in the discussion is referring to electron degeneracy pressure, which only manifests itself in extreme examples such as the core of a collapsing star. The electron degeneracy pressure between your feet and the floor is negligible. Someguy1221 (talk) 03:22, 6 November 2012 (UTC)[reply]

(This is incorrect—see below.) -- BenRG (talk) 23:13, 6 November 2012 (UTC)[reply]

This is what he said, "The electrostatic force behaves completely differently from the contact forces we're used to. The floor pushes back exactly as much as you push on it; contrariwise, the electrostatic force's push depends entirely on the distance. You can derive Hooke's Law from the basic kinematics of QM - the compressing of the states by Pauli exclusion causes a corresponding increase in momentum as per the uncertainty principle. I'm sorry to break it to you but you've been taught a wrong fact for a long time." ScienceApe (talk) 03:32, 6 November 2012 (UTC)[reply]

From our article: A material subjected to ever increasing pressure will become ever more compressed, and for electrons within it, the uncertainty in position measurements, Δx, becomes ever smaller. Thus, as dictated by the uncertainty principle, the uncertainty in the momenta of the electrons, Δp, becomes larger.. In short, your friend doesn't know what he's talking about. The pressure exists as he suspects, but its value is insignificant outside of extremely dense matter. Someguy1221 (talk) 03:42, 6 November 2012 (UTC)[reply]
Also note that "The floor pushes back exactly as much as you push on it" is true for any explanation of a static situation. --Stephan Schulz (talk) 06:56, 6 November 2012 (UTC)[reply]

Solids and liquids are incompressible because of electron degeneracy pressure, and you don't fall through a solid floor (as opposed to a liquid floor) because of the intermolecular bonds. This has been asked before on the ref desk. -- BenRG (talk) 07:03, 6 November 2012 (UTC)[reply]
Yea Newton. The third law applies here: The weight of an object pressing upon another is countered by an equal and opposite force exerted by the electrostatic interaction between electric fields of atoms. Plasmic Physics (talk) 21:12, 6 November 2012 (UTC)[reply]
No, as I said above, it's electron degeneracy pressure that presses upwards on your feet, though it is horizontal electrostatic tension that prevents the floor from breaking apart under your weight. Newton's third law applies everywhere, but doesn't explain why you don't fall through the floor. You would fall through a floor made of jello, though the third law applies there too. -- BenRG (talk) 23:13, 6 November 2012 (UTC)[reply]
I just don't buy it, Ben. At normal densities, long-distance molecular interactions can be explained just fine without invoking degeneracy pressure. I don't see a need for it in this situation. Someguy1221 (talk) 00:58, 7 November 2012 (UTC)[reply]
Exactly. Where does compressability of solids fit into the degeneracy idea? Plasmic Physics (talk) 01:48, 7 November 2012 (UTC)[reply]
I would be surprised if it was correct, it would turn my understanding of atomic-force microscopy upside-down. Plasmic Physics (talk) 01:51, 7 November 2012 (UTC)[reply]

There is a bit of confusion above. The Pauli exclusion principle gives rise to exchange interactions. For fermions, such as electrons, this quantum mechanical effect manifests as an intrinsic repulsion towards overlapping orbitals. This exchange interaction is absolutely critical in determining the bond length in all forms of chemical bonding. As such, the Pauli exclusion principle plays a key role in determining the distance between atoms in all forms of solids, and by extension influences many properties, such as rigidity. Without the exchange interaction, solids wouldn't be solid at all. At the same time, the electrostatic attraction between the positively charged nuclei and negatively charged electrons is also essential to chemical bonding and determining the properties of solids. Without the electrostatic forces there would also be no solids at all. You won't get an accurate model of solids unless you consider both electrostatic interactions and exchange interactions. See, for example, the Lennard-Jones potential model which gives an example of how repulsive exchange interactions and attractive Van der Waals forces (electrostatic), combine to create a local minimum in the separation potential. Its that local minimum that creates a natural bond length. When you push on a solid, the atoms in your hand displace the atoms in the solid. That push translates into the solid by stretching some of the bonds and compressing others, and ultimately it is that displacement from the ideal bonding length that causes the solid to exert a collective force that pushes back against you. This is much the same as if you could imagine the solid having been made of many tiny little springs. Since the chemical bonding would not occur without considering both principles, I would say that both electrostatic forces and the Pauli exclusion principle are necessary to understanding why we are able to stand on the floor. Dragons flight (talk) 03:18, 7 November 2012 (UTC)[reply]

People talking alone

What's wrong with these people? I know that they are not necessarily mentally ill, some just take drugs, also legal drugs like coffee. But, what has happened in their brain, what is the brain mechanism of "mental talk, but not aloud"? Comploose (talk) 00:59, 6 November 2012 (UTC)[reply]

Please see Intrapersonal communication. Specifically: "Simon Jones and Charles Fernyhough cite research suggesting that our ability to talk to ourselves is very similar to regular speech. This theory originates with the developmental psychologist Lev Vygotsky, who observed that children will often narrate their actions out loud before eventually replacing the habit with the adult equivalent: sub-vocal articulation. During sub-vocal articulation, no sound is made but the mouth still moves. Eventually, adults may learn to inhibit their mouth movements, although they still experience the words as "inner speech"."A8875 (talk) 01:40, 6 November 2012 (UTC)[reply]
It can also be helpful in some cases, like repeating something out loud to remember it. I believe this trick works because more of the brain is involved in speaking and hearing the words than just thinking them. There's also the issue of people who aren't able to read or count silently. This could, indeed, in some cases compensate for a deficiency in their brain, by using different neural paths to bypass the non-functional portion. StuRat (talk) 01:44, 6 November 2012 (UTC)[reply]
what is the brain mechanism of "mental talk, but not aloud" - How else does one think, except as mental talk, but not aloud? All my thoughts that are not pictures are words, sentences, paragraphs, even dialogues. How else do you reach conclusions, consider options, assess situations, except as words not spoken aloud? Have I missed something? There are people who process things externally. I worked with one who drove most of us into any room in the building where she could not be heard. Is that what you mean? Bielle (talk) 02:03, 6 November 2012 (UTC)[reply]
I wanted to contrast people who are "mental talk, in silence" to others who are "mental talk, aloud". Something is different in the second case: too much coffee, stress, some drugs sometimes makes some people be in the second group, even if they are normal under other aspects. I was not asking about any contrast in terms of "mental talk" / "no mental talk". Comploose (talk) 02:19, 6 November 2012 (UTC)[reply]
My mom always used to say "People who talk to themselves are either crazy or have money in the bank." To which she would always add "And I'm flat broke." Of course, it isn't any sign of being mentally defective in any way to occasionally talk to yourself aloud. People who aren't under the influence of drugs or stress or who are otherwise perfectly normal, mentally (for any given standard of "normal"), do sometimes speak to themselves out loud. Of course, people carry on all sorts self-talking, whether it is internal monologue, keeping a diary or journal of some sort. One journal articles I found on actual talking out loud to oneself is [3].--Jayron32 03:35, 6 November 2012 (UTC)[reply]
Unfortunately, when I engage in an internal monologue, I always end up heckling myself. StuRat (talk) 03:23, 6 November 2012 (UTC) [reply]
At least that won't make you go blind. Be grateful for the respite.  :) -- Jack of Oz [Talk] 05:18, 6 November 2012 (UTC) [reply]
Isn't heckling yourself by definition internal dialogue? -- Q Chris (talk) 13:47, 6 November 2012 (UTC)[reply]
Maybe we should all get in touch with our inner StuRat and have a good old chinwag. I'll get round to it when I've finished my current vitally important project (transplanting Mt Everest to the Antarctic a teaspoon at a time). -- Jack of Oz [Talk] 21:26, 6 November 2012 (UTC) [reply]
"I used to be poor and crazy, but now I'm rich and eccentric." Someguy1221 (talk) 03:19, 6 November 2012 (UTC)[reply]
It's entirely natural to talk out loud, whether there is someone there to listen or not. "How can I know what I think until I hear myself say it?" The discipline of speech is good for ordering internal thoughts. --TammyMoet (talk) 10:13, 6 November 2012 (UTC)[reply]
what's wrong is that nobody's given them an old dead cell phone. if they held the phone (or even a toy phone, or anything of similar form factor, really) up to the side of their face, they could talk to themselves all they want and nobody would notice. Gzuckier (talk) 02:06, 9 November 2012 (UTC)[reply]

Nature of sound

If a car crashes in the forest and nobody is there to hear it, does it still make a sound? Bonkers The Clown (talk) 07:53, 6 November 2012 (UTC)[reply]

Yes. HiLo48 (talk) 08:03, 6 November 2012 (UTC)[reply]
And what makes you say so? Can you elaborate as to why "yes" and not "no"? Bonkers The Clown (talk) 08:05, 6 November 2012 (UTC)[reply]
The Laws of physics. HiLo48 (talk) 08:14, 6 November 2012 (UTC)[reply]
Well the driver would presumably hear something. But this is essentially the same question as If a tree falls in a forest, where this is discussed. (I changed the title of your question to something more meaningful.)--Shantavira|feed me 08:27, 6 November 2012 (UTC)[reply]
This depends on your definition of "sound". The way physicists define it, as a pressure wave, does not require an observer. Indeed, we would need to change much of physics to allow for a collision, in air, which does not produce pressure waves. Now, if a philosopher wants to define sound as only existing when it is heard, that's fine, but has nothing to do with science. StuRat (talk) 19:55, 6 November 2012 (UTC)[reply]
What if it is a superpositional Schrodinger soundwave? Plasmic Physics (talk) 20:59, 6 November 2012 (UTC)[reply]
In that case, only dogs can hear it. And Schrödinger. ←Baseball Bugs What's up, Doc? carrots23:20, 6 November 2012 (UTC)[reply]
And potentially dead cats. StuRat (talk) 03:21, 7 November 2012 (UTC) [reply]
I mean, if the crash was initiated by a quantum trigger, would it still make a definitive sound if there is no observer? Assuming unrealistically perfect conditions. Plasmic Physics (talk) 01:42, 7 November 2012 (UTC)[reply]

protons

Is the proton a fundamentally stable particle? Or does it have a finite lifetime as predicted by some extensions to the standard model? What explains the anomalous spin of protons? :) Bonkers The Clown (talk) 08:39, 6 November 2012 (UTC)[reply]

If you're aware that physicists can't decide whether the proton is fundamentally stable, why would you bother asking a bunch of random people on the internet? Someguy1221 (talk) 09:47, 6 November 2012 (UTC)[reply]
Additionally, our article on the Proton spin crisis mentions that this is an unsolved problem, which I'm sure you were already aware of. Someguy1221 (talk) 09:51, 6 November 2012 (UTC)[reply]
A lower limit for the half-life of the proton is 2.1 × 1029 a, but proton decay has never been observed. Double sharp (talk) 14:05, 6 November 2012 (UTC)[reply]

The proton is almost certainly unstable, the only question is how large the half life is. Hawking has argued that the proton would at least have to decay via making a transition to a virtual black hole which then evaporates via the Hawking process.

Also note that the deuteron is known to be unstable in the standard model with a half life of about 10^(218) years. Count Iblis (talk) 16:18, 6 November 2012 (UTC)[reply]

Proton decay does or should have the relevant ideas covered. DMacks (talk) 16:29, 6 November 2012 (UTC)[reply]

Worried to death?

I was thinking about a phrase my mother used to use "I was worried to death!" and wondered if, in fact, it was possible to shorten your life - or at least adversely affect it - by worrying! Is there any good scientific research on the deleterious effects of worry on human beings? I don't mean the mental illness anxiety, by the way. Just plain, old fashioned worry. --TammyMoet (talk) 10:15, 6 November 2012 (UTC)[reply]

Worrying causes negative thoughts and negative thoughts translate into depression. And you'll die sooner or later if you worry daily. Don't worry. Be happy. :) Bonkers The Clown (talk) 13:38, 6 November 2012 (UTC)[reply]
Citation needed. AlexTiefling (talk) 13:41, 6 November 2012 (UTC)[reply]
If you mean worry v.tr. 3 a (To seize with the teeth and shake or tug at repeatedly) then you certainly could be worried to death rather quickly! -- Q Chris (talk) 13:44, 6 November 2012 (UTC)[reply]
Topical question, apparently empirically answered this week in a rather tragic death. well, okay, it was 'heartbreak', but that has to be a near-cousin of worry, non? --Tagishsimon (talk) 14:59, 6 November 2012 (UTC)[reply]
That looks like hearsay to me. AlexTiefling (talk) 15:05, 6 November 2012 (UTC)[reply]
Here's a news article, with links to a scientific article [4]. It explains how grieving can greatly increase incidence of heart attack. I don't know if "grieving" is similar enough to "worry" for your purposes, but there it is... SemanticMantis (talk) 17:18, 6 November 2012 (UTC)[reply]
Not grieving? Keeping it all inside? ←Baseball Bugs What's up, Doc? carrots23:18, 6 November 2012 (UTC)[reply]
Stress certainly has negative effect on health, and thus can reduce your lifetime. Not sure if that's what you mean by worry. - Akamad (talk) 17:41, 6 November 2012 (UTC)[reply]
To clarify, what I meant by "worry" is something like sitting envisioning a future event that might or might not actually happen and all its ensuing ramifications. You know, the sort of unproductive thought processes you indulge in at 3 am... --TammyMoet (talk) 20:47, 6 November 2012 (UTC)[reply]
As with "Future Events Appearing Real". ←Baseball Bugs What's up, Doc? carrots23:18, 6 November 2012 (UTC)[reply]
Or "False Evidence Appearing Real". Yeah that kind of thing. --TammyMoet (talk) 10:21, 7 November 2012 (UTC)[reply]
The proverb has it "worry is interest paid on a debt that may never come due". On the other hand, there is another hand. The flip side is that sometimes worry concentrates the mind to the point that you find an actual and effective contingency plan to deal with the potential problem.
So I'm unwilling to say that worry is a uniformly bad strategy — it's saved my butt more than once. It does have very real costs, though, and they have to be balanced against the potential benefits. How you do that is your worry. --Trovatore (talk) 10:26, 7 November 2012 (UTC)[reply]

how to be more powerful (literally, not philosophically)

EDIT: First of all this question had been misinterpreted. This is not a philosophical question but purely practical. I hope you are able to follow my reasoning, and what the question is.

Let us begin with the observation that with a coffee mug or any other object in front of you (try it!) you can cause the coffee mug to move simply by selecting a new location for it, picking it up, and moving it.

The proximate cause of the move or displacement was your own thought. Since, for example if you moved it with your right hand, if after that thought you found your right hand encumbered by something you forgot about, you would have just moved it with your left hend. It is really your thought that caused the change.

So, a thought is a very powerful thing.

But, on the other hand, there are thoughts that do not cause a change in the world. They come and go, like an old man's erection. They affect nothing.

I would like to know how to make my thoughts more powerful. To begin with, by increasing their effectiveness on my own body, and then on my team, organization, country, world. What do you suggest is an appropriate way to increase the leverage of my thought, so that I can not only move a coffee mug, but reshape my life, country, or world? Please be detailed.--91.120.48.242 (talk) 13:19, 6 November 2012 (UTC)[reply]

Begin by making good use of your time. Spend less of it asking unproductive questions on RefDesk.
In all seriousness - economic privilege will make more difference to the 'power of your thoughts' than anything else. If you're white, male, American, upper- or middle-class, able-bodied, neurotypical, not subject to mental ill-health, heterosexual, and not too outspoken about your religious views, you'll go far. As to what you can do, yourself - there are many things, but the degree to which they're effective correlates to the extent that you match the privileged profile described. (Obviously this is socially determined; if you're Japanese rather than American, then being ethnically Japanese rather than white is the appropriate factor, etc.) AlexTiefling (talk) 13:34, 6 November 2012 (UTC)[reply]
I really did not mean on this level, I meant on an everyday level, hence the example of a coffee mug. I mean things like translating from "having a product idea" to actually shipping it, much like translating from "I want to move this cup over" to it actually being moved over. Can you address it in more practical terms? --91.120.48.242 (talk) 14:10, 6 November 2012 (UTC)[reply]
If you mean literally then a Powered exoskeleton would do it for you. -- Q Chris (talk) 14:25, 6 November 2012 (UTC)[reply]
That must be one massive mug. Plasmic Physics (talk) 20:53, 6 November 2012 (UTC)[reply]
You talked about your ability to "reshape my life, country, or world" - that sounds like going beyond the immediately physical effect of your thoughts. 'Reshaping the world' doesn't sound like 'an everyday level' to me. If you do mean things like translating product ideas to manufactured products, then my earlier comments still stand. AlexTiefling (talk) 15:04, 6 November 2012 (UTC)[reply]
Try reading How to Win Friends and Influence People by Dale Carnegie. StuRat (talk) 19:50, 6 November 2012 (UTC)[reply]
or Power for Success by Frank Haddock. Also speaking the truth can garner other's support. and being able to find hidden truths is very powerful as it is difficult to deny/ignore GeeBIGS (talk) 01:19, 7 November 2012 (UTC)[reply]
If you just want to be "powerful" in the sense of "changing the world", try burning down a world-famous landmark. That worked pretty well for Herostratus, who sought fame by burning down the Temple of Artemis (and succeeded). If you want to be powerful in a "good" way, try political campaigning, running for office, founding a charity organization, or joining the Syrian rebels. All of these things have a good chance of making a non-negligible impact on the world. --140.180.252.244 (talk) 02:06, 7 November 2012 (UTC)[reply]

If we look at just watts of energy that are created or responsible for, I'm pretty sure the founder of Ikea was far, far more powerful by founding ikea than by donning an exoskeleton and doing work himself instead of getting ohters to do it. In monetary terms, there is no real way to get as much value out of an exoskeleton as by mobilizing loads of people in the form of a company either. So on a long-scale in physical terms (watts of work performed) or economic terms (value produced) you can beat an exoskeleton. I would be interested in how to start that process. The links given above about "Power for success" or the other link are an interesting idea, but really you are telling me, "Put on your to-do list: buy and read Power for success." I believe there are more effective sentences you can tell me, as I probably won't buy and read that book. What can I do today that will measurable impact my ability to follow through on my ideas? --91.120.48.242 (talk) 15:19, 7 November 2012 (UTC)[reply]

I actually thought in this vaguely general direction, back in my psych major days... it occurred to me as you say, that our basic thought is not "lift your right arm, advance it towards the cup, lower it and grasp the cup" etc., it's "move the cup", and as you say, if your right hand is blocked, you move your left, etc. If both are blocked, and you ae sufficiently motivated, as by somebody holding a gun to your head and demanding that you move the cup, you will move it with your feet, or your teeth, or nudge it with your nose, or whatever. So it seemed to me that the basic thing is goal-orientedness of thought. And i now enlarge on that to answer your question somewhat vaguely is the way to empower your thoughts in that sense is, therefore, to keep your eyes on the ultimate goal you have in mind, and not to get too tied down to a specific pathway or procedure, etc. Rather than just running up against a block and grinding away at it, find a way around or over it. Gzuckier (talk) 02:18, 9 November 2012 (UTC)[reply]

Air quality in Prometheus

During landing (minute 23 and about three quarters) on the foreign planet, the readings of the gases comprising the atmosphere are given as 79% nitrogen, 21% oxygen and trace argon -- something very similar to earth's atmosphere. But a scientist on board then says that CO2 readings are three times what they would be on earth -- about 3% -- and she explains that one would die after only a few minutes of exposure to such an atmosphere without a protective suit (and, I'm assuming, a breathing device). Why would triple the amount of CO2 necessitate a protective suit? DRosenbach (Talk | Contribs) 14:35, 6 November 2012 (UTC)[reply]

Not surprisingly our article Carbon dioxide#Toxicity mentions some of the effects of an increase carbon dioxide:

In concentrations up to 1% (10,000 ppm) will make some people feel drowsy.[78] Concentrations of 7% to 10% may cause suffocation, manifesting as dizziness, headache, visual and hearing dysfunction, and unconsciousness within a few minutes to an hour.

It also links to Hypercapnia, which is our article on the condition of excessive carbon dioxide in the blood. 3% seems to be in the range of starting to get close to dangerous but perhaps not quite there yet and I'm not sure that dying within a few minutes is likely. I'm not even sure it's likely at 7% although I would't want to risk it particularly if anyone who could try to save me is going to be exposed to the same conditions. Perhaps the producers were thinking of carbon monoxide?
BTW our article also mentions something which you can check in Atmosphere of Earth. Either your memory of the movie is wrong, or there's something seriously wrong with it. 3% carbon dioxide is not triple the concentration of earth (which isn't 1%, that's well over 1 order of magnitude wrong).
Nil Einne (talk) 14:47, 6 November 2012 (UTC)[reply]
You're right -- all she said was "[it's only like earth] if you're breathing through an exhaust pipe. CO2 is 3%. Two minutes without a suit and you're dead." DRosenbach (Talk | Contribs) 14:50, 6 November 2012 (UTC)[reply]
The exhaust pipe bit makes me think it even more likely the producers were confusing carbon monoxide and carbon dioxide. Nil Einne (talk) 14:52, 6 November 2012 (UTC)[reply]
This source also mentions [5] that the planet in question was most likely supposed to be carbon monoxide as it's been suggested before that the planet is polluted with carbon monoxide. (I'm not entirely sure why that would be the case, if the planet has similar atmospheric conditions and temperature to earth, you would expect any carbon monoxide would be oxidised to carbon monoxide so there needs to be a constant high source or some reason why this is not happening, see Carbon monoxide#Atmospheric presence. You could get something like Venus where photodissociation produces carbon monoxide but I'm not entirely sure how likely that is if you otherwise have similar conditions to earth. At the very least the high levels of UV may be an additional concern.) Nil Einne (talk) 15:02, 6 November 2012 (UTC)[reply]
Using the figures in Hypercapnia, 10kPa will cause acute symptoms shortly leading to death--that's about a 10% concentration. Chronic respiratory acidosis will set in at about half that concentration. The 3% number is probably global warmist propaganda to make CO2 sound more dangerous than it is. CO2 and CO are odorless. Given how bad that movie stank, I suspect they meant to say it had a 3% concentration of flatus. μηδείς (talk) 15:02, 6 November 2012 (UTC)[reply]
Don't assume malice when stupidity will suffice.-- OBSIDIANSOUL 16:45, 6 November 2012 (UTC)[reply]
All of the above explains why you would need breathing apparatus; but not why a full-body suit would be needed. One reason would be solar radiation or protons, but as space exposure notes, that's a long term concern, not a 3-minute-death issue. Another would be very low pressure - if the ambient pressure was very low (cf Armstrong limit) an explorer would develop ebullism (as Kittinger did on his hand during his record dive), even if she was breathing air at 1 bar. But the planet exhibits a dense cloud layer miles above the surface, suggesting the surface pressure is pretty high. So that leaves the possibility that the atmosphere contains something so unpleasant that even a few minutes of it touching exposed skin would be fatal. Skin is pretty good at keeping out the environment, so that would necessitate the atmosphere was significantly acidic, caustic, or otherwise significantly toxic - something much worse than exhaust gas. Under relatively normal pressure and the specified air contents, they'd just need a respirator - which is exactly what the human characters wear in Avatar. I fear the suits are there for the theatrical "let's suit up" effect they provide. -- Finlay McWalterTalk 16:58, 6 November 2012 (UTC)[reply]
And breathing specifically is the express purpose of the suits - Holloway asks David why he's wearing a suit when "you don't breathe" (0:25:38) - not pressure, toxicity, or biosecurity. -- Finlay McWalterTalk 17:05, 6 November 2012 (UTC)[reply]
Actually, about %5 carbon dioxide is needed to cause chronic acidosis, so short exposures of 3% to healthy people should be harmless. The purpose of the suits was to show the contrast between the inside and outside environments, to give the alien something to break through--to allow the use of the suits as props. μηδείς (talk) 18:05, 6 November 2012 (UTC)[reply]

Note that over about 6bar, 80% nitrogen will give nitrogen narcosis, and over about 9bar, 20% oxygen will cause oxygen poisoning. It is the partial pressure that is important - the pressure mutltiplied by the concentration. CS Miller (talk) 20:54, 7 November 2012 (UTC)[reply]

  • Hmmm. It's possible to overcome acidosis by infusion of sodium carbonate [6], the kidney tubule can regulate ammonia production [7], and damage to the inner ear can reduce the increase in ventilation [8]. I would not rule out the possibility of protecting people from moderate CO2 levels by some sort of medication - I'm not even convinced that gradual adaptation to this environment is not possible, unless and until I find data to prove it. Wnt (talk) 16:31, 8 November 2012 (UTC)[reply]
Lemme get this straight. A film which identifies "35 light years" as "a half billion miles", and you expect them to understand physiological effects of CO2 at above an average third grader's comprehension level? Gzuckier (talk) 02:51, 9 November 2012 (UTC)[reply]
Gzuckier has the right of it I think. Scripts don't have to make sense scientifically to make money. — Preceding unsigned comment added by 67.188.225.212 (talk) 08:00, 9 November 2012 (UTC)[reply]

Alcoholism - Biological process

Hi all, First, please note that this is not a request for medical advice, but a question about specific biological process inside the body. Please do not give any specific medical advice. I have read places that alcoholism/recovering from alcoholism can cause the following 2 effects:

1: Extremely bad body odor 2: External bruises around the liver, external scarring.

I was wondering how it is that the body/the alcohol ingested can cause these problems - and what is the chemical/biological reason for this on a fairly simple level. There doesn't seem to be anything lited in the article on Alcoholism. Please do not give advice on how to treat these as it consitutes medical advice and I don't want my post deleted. 80.254.147.164 (talk) 15:08, 6 November 2012 (UTC)[reply]

  1. Alcoholism causes a deterioration in self-care, drunks don't shower as regularly as non-drunks in a given population.
  2. Alcohol impairs blood clotting - bruising anywhere on the body may be exacerbated. Roger (talk) 18:14, 6 November 2012 (UTC)[reply]
1) Alcoholics may also sweat alcohol, which smells unpleasant to most, and also readily evaporates, taking regular body odor with it. StuRat (talk) 19:45, 6 November 2012 (UTC)[reply]
  • On odor I found what might be a useful reference [9] - I'll leave it to you to track it down.
2) Alcohol can act as a blood thinner, which can cause blood to leak out of blood vessels and cause bruising. StuRat (talk) 23:34, 7 November 2012 (UTC)[reply]

Death determination in Sherlock Holmes

Maybe we're all just a little too informed these days because of shows like CSI, but shouldn't Watson have been confused by the hanged man's lack of rope marks on the neck that usually occur during a hanging. It seems awfully sloppy for him to have overlooked such a thing, especially since both he and Holmes are so astute in so many other scientific findings? I mean, sure, it would disrupt the entire plot, but it seems to undermine it just a tad too much -- they could have had him executed in another fashion to preserve his apparent death and subsequent resurrection. DRosenbach (Talk | Contribs) 14:54, 6 November 2012 (UTC)[reply]

It was somewhat sloppy, but Watson was only there to pronounce death, not perform an autopsy or otherwise determine cause of death. IIRC, he checked for pulse and maybe made a mirror test, but that was it. The guy who cut Blackwood down should have been the one to mention the oddity, I would think. Matt Deres (talk) 15:35, 6 November 2012 (UTC)[reply]
The entire premise of Sherlock Holmes is faulty, that every clue leads you down a single path. For example, "no sign of a break-in" = inside job. Instead, every clue only alters the probabilities, and you never can absolutely conclude anything, only that certain chains of events are more likely than others. StuRat (talk) 19:42, 6 November 2012 (UTC)[reply]
How can the premise of investigaion be faulty? Holmes and Watson are merely before their time, applying CSI style crime investigation to the late 19th century. Sure, licking the broken fragments of a fractured stone tomb cover and examining the premises of potential criminal activity without a warrant is not something that would be done today...but I still think it ought to be considered realistic fiction. DRosenbach (Talk | Contribs) 22:02, 7 November 2012 (UTC)[reply]
The premise is that there are only a small number of possibilities and that every clue eliminates some of them. This is "absolute logic", but is not really what applies in the case of criminal cases. It's more "fuzzy logic", where each clue only pushes you a bit more or less in one direction. Even seemingly objective results, like fingerprints, are really still subject to opinions, falsification, etc. And they really stretch it in the case of Holmes' deductions. For example, he once concluded that a man with a tan in England during winter had just returned from India. Well, that's one way to get a tan, but there are many other ways. StuRat (talk) 23:37, 7 November 2012 (UTC)[reply]

Cloning in animals Vs. plants

A gene of interest has been isolated via restriction enzyme digest and amplified using polymerase chain reaction (PCR). We want to clone this gene of interest by inserting it into a plasmid vector, making a recombinant DNA molecule. What differences in the cloning method would we want if we wanted to express the gene of interest into a plant? — Preceding unsigned comment added by 150.203.114.11 (talk) 15:02, 6 November 2012 (UTC)[reply]

I'm sorry, but this (to me) a very general and vague question. Can you clarify? In case it helps, one simple difference between cloning in animals and plants is that many plants naturally clone themselves, see vegetative reproduction. In animals, natural asexual reproduction is highly restricted, mostly to little things like hydras. How is this relevant to gene insertion? SemanticMantis (talk) 17:10, 6 November 2012 (UTC)[reply]
Maybe some context will help. That question is the last in a sequence. Here it is:
A gene of interest has been isolated via restriction enzyme digest and amplified using polymerase chain reaction (PCR). We want to clone this gene of interest by inserting it into a plasmid vector, making a recombinant DNA molecule.
What features of a plasmid are necessary for this molecular cloning to work?
How do we get the gene of interest into the plasmid vector?
The gene of interest is inserted into the plasmid. Describe two methods by which the plasmid can be transformed into E.Coli.
How do we know which transformed E.Coli cells are the desired clones?
Describe any differences in the cloning method if we wanted to express this gene of interest into a plant.
--150.203.114.11 (talk) 17:16, 6 November 2012 (UTC)[reply]
Ok, that looks like homework. If you want more help, you'll have to explain how you've started, what sources you've looked at, and where you are stuck. We generally have a rule against doing people's homework for them, see the disclaimer at top. SemanticMantis (talk) 17:20, 6 November 2012 (UTC)[reply]
It's not homework; it's pracice questions for the examin. And besides, this is biology; there is no such thing as "being stuck" like there is with maths or physics; you either know the answer a priori or you don't; it's not something you can deduce or work out.--150.203.114.11 (talk) 17:31, 6 November 2012 (UTC)[reply]
Thank you for being honest. Practice examination questions are clearly a form of homework. If we work out the answerr for you, that helps us get smarter, but doesn't help you. Your claim about being stuck is nonsense. Floda 121.221.77.87 (talk) 00:26, 7 November 2012 (UTC)[reply]
To help you work it out: "A gene of interest has been isolated via restriction enzyme digest and amplified using polymerase chain reaction (PCR). We want to clone this gene of interest by inserting it into a plasmid vector, making a recombinant DNA molecule. What differences in the cloning method would we want if we wanted to express the gene of interest into a plant?" I should add that I've never actually done this to a plant and I'm rather out of date in my recollections off-hand, so I'll leave it to you to mull over your homework and hopefully provide the latest updates to Bacterial_transformation#Plants, Plant_virus#Well_understood_plant_viruses, gene gun, etc. as you make your observations. Wnt (talk) 23:14, 7 November 2012 (UTC)[reply]

American Bison Furs

With the Ameican Bison once numbering 30 to 60 million, and hundreds of thousands to millions of hides being shipped back to the east coast every year in the second half of the 1800's, what happened to all the furs, rugs, robes, etc...? A majority would be lost, worn out, thrown away, but with untold millions of hides processed into some sort of product shouldn't they still be a realitively common item in the United States? Or do treated hides just not last long enough? Beach drifter (talk) 15:44, 6 November 2012 (UTC)[reply]

Would not much of the hides be turned into leather? I'd guess 100+ year old shoes, suitcases, etc, are relatively common, but equally that the source - bison, or cow, for instance - less obvious. --Tagishsimon (talk) 16:00, 6 November 2012 (UTC)[reply]
I don't think furs last all that long, at least if you use them. If stored in a refrigerator for a century they last, sure, but not many people did that. StuRat (talk) 19:36, 6 November 2012 (UTC)[reply]

What is a major disadvantage of PCR compared with in-vitro replication?

What is a major disadvantage of PCR compared with in-vitro replication?--150.203.114.11 (talk) 16:50, 6 November 2012 (UTC)[reply]

What did your teacher say was a major disadvantage, or what was written in your textbook? That's where you will find the answer to your question. --Jayron32 18:40, 6 November 2012 (UTC)[reply]
Isn't PCR a form of in-vitro replication? DRosenbach (Talk | Contribs) 22:04, 7 November 2012 (UTC)[reply]

Science fiction question regarding physics

I'm hoping to get a realistic physics viewpoint on a fictional event. Say there is a person who is, for one reason or another, trapped between two different timelines. Their body rapidly phases between the two, leaving a vacuum during the infinitesimally short time they are not present. Would such a vacuum be extremely cold? For directly, would the person be cold to the touch? Thanks in advance for anyone who answers. --Ghostexorcist (talk) 17:56, 6 November 2012 (UTC)[reply]

The process you are describing would cause the conservation of matter and energy to be violated in each timeline. This is such a wild deviation from normal thermodynamics, I don't think we can easily speculate. But how rapid is the oscillation? Will Brownian motion convey air molecules into the vacated space prior to the person returning? Because if so, that space might be expected to get very hot, rather than cold - with consequent danger to the oscillating person, not just bystanders. AlexTiefling (talk) 18:01, 6 November 2012 (UTC)[reply]
Well, just as physics was modified to allow for the conservation of total mass and energy, versus each separately, let's imagine that it's again modified to allow for conservation of the total mass and energy in all possible timelines. In this case, assuming the person does not materialize in a vacuum, the air where they appear needs to go somewhere. The most elegant solution is to send that air the other way, so no vacuum is created. If, instead, you suddenly blow the air out of the way, then that will create a minor explosion where the person arrives, and a sudden implosion where they left. Both would result in an increase in temperature. The place they left would have a net reduction in air pressure and the place where they arrived would have an increase. In an enclosed space, this could pop your ears. If you are the person being transported, this might pop your ears in any case. So, sending the air back the other way is the best solution (although this could introduce bacteria, pregnant mosquitoes, and such into the other environment). StuRat (talk) 19:29, 6 November 2012 (UTC)[reply]
This popping is the option used in The Stars My Destination, by the way. Wnt (talk) 15:58, 7 November 2012 (UTC)[reply]
This is just basically nonsense. Any one contradiction logically implies all other contradictions. You might as well ask if 2=3, then does 5=6? The answer can be yes (if 2=3, then 2+3=3+3, or no, because 2x2=3x2, i.e. 4=6, not 5. Or does it? It is utter bullshit, so make up whatever ad hoc nonsense (like air switching) you want. μηδείς (talk) 19:50, 6 November 2012 (UTC)[reply]
Medeis put it a bit impolitic, but she's essentially correct. It was stated much better by Arthur C. Clarke, who had Clarke's three laws, the third of which is very germane to the question. What Clarke said was "Any sufficiently advanced technology is indistinguishable from magic." Which is to say that, when writing speculative fiction of any sort (either fantasy or sci-fi), it isn't really necessary to be entirely correct with regards to the laws of physics, indeed it is better to not even try at all, since any half-assed kludge attempting to explain FTL communication or teleportation or any of a number of other common sci-fi tropes will necessarily invite criticism (justly so) as being completely incompatible with the known laws of physics. So Clarke's solution was to just not try: let the magic be magical. Also relevent: when writing about the present, we don't spend any time explaining TV or the internet or any of a number of other technologies that would have seemed quite "magical" to anyone living, say, 300 years ago. We all know TV works, and many of us know why it works, and even if we don't it is common enough to be accepted without an exhaustive discussion of how the laws of physics allow it to work. In the same way, there's no need to make room in your science fiction writing to allow for lengthy explanations of how physics allows for some bit of sci-fi technology: the characters in the story will accept it, so there's no real need to explain it to the readers, and again, any attempt to explain it to the readers will only come off as clumsy and essentially wrong. --Jayron32 20:06, 6 November 2012 (UTC)[reply]
I disagree. I like my sci-fi to at least be within the realm of possibility. Take the Stephen King book The Stand. It started out about something possible, an out-of control disease spread from a germ warfare lab, then became an absurd tale about the devil. At that point I lost interest. Also note that real sci-fi occasionally gets it right, with Clarke predicting communication satellites and even The Jetsons predicting the microwave oven. StuRat (talk) 20:19, 6 November 2012 (UTC)[reply]
De gustibus non est disputandum. Clarke has sold a book or two in his day, and may have something relevent to say on writing successful science fiction. --Jayron32 20:22, 6 November 2012 (UTC)[reply]
Not to mention that The Stand really isn't science fiction, at least not the type of "spaceships and robots" science fiction that both the OP and Clarke are discussing here. Applying his law requires sufficiently advanced technology, and absolutely nothing in The Stand is really advanced technology. It is essentially set in the 1970s. Finally, not liking Stephen King is fine: many people don't like him. But not liking The Stand merely because you didn't expect anything supernatural in it seems incongruous. The supernatural is kinda his thing. I mean, even assuming you read The Stand the year it came out, there were previous books about A telekinetic teenager, a town of Vampires, and a telepathic child and a haunted hotel. It took him until Cujo before he wrote a book entirely devoid of the supernatural, and even after that, the vast majority of his books feature it prominently. --Jayron32 21:12, 6 November 2012 (UTC)[reply]
I have to agree with Jayron re The Stand. King almost always writes fantasy dramas. The Stand succeeds briliantly as a Drama. (I am not a huge fan of King's, but his good books are good.) The Tommyknockers is pure science fiction, and it's crap. 11/22/63 is almost pure science fiction, and succeeds brilliantly. King has a thing for ghosts or devils opening doors. he does it in The Shining and The Stand, which are both successful dramas. He needs to be judge within universe and onthe basis of a drama writer.
If you want hard sci-fi read the early Larry Niven. For example, his teleportation boothes are subject to the conservation of momentum and require oceanic buffers to prevent zoom-splatification. As far as I am concerned, a good human conflict is the essence of drama. If you want good sci-fi then look for good drama. Miven could do this and Heinlein was the best. No one ever gave a whit about the Dean's bull ...shit. μηδείς (talk) 21:48, 6 November 2012 (UTC)[reply]
I liked Heinlein's supernatural stuff as much as his hard SF. I mean the early supernatural stuff, like Gulf and The Unpleasant Profession of Jonathan Hoag. Stranger is OK but it's on the border of where he abandoned tight plotting for having his characters explain things to each other. --Trovatore (talk) 21:57, 6 November 2012 (UTC)[reply]
Other good examples: Lost Legacy, Waldo, Magic, Inc. --Trovatore (talk) 23:24, 6 November 2012 (UTC)[reply]
One of the things to remember about good science fiction writing is that it is good writing first and good science fiction second. What makes some of the best science fiction is the sort of universality of themes and novel approaches to them, which isn't something that is strictly confined to science fiction, but is what makes all quality literature worth reading. In good science fiction, the "science fiction" elements blend in or melt away in ways that make them part of the background of a really good story. Which isn't to say that such elements are unimportant, but there's a certain MacGuffin-like quality to science fiction: If you strip away the sci-fi bits and still have a good story and compelling characters, and if the sci-fi itself doesn't distract from it, you have a great book. I mean, what is The Moon is a Harsh Mistress if it isn't a work of revolution fiction or a critique of colonialism. How is the theme of A Space Odyssey all that different from the Genesis narrative of Adam and Eve in the Garden of Eden? Isn't the Monolith merely the Tree of Knowledge? Isn't the Foundation Trilogy basically a fanciful version of Gibbon's Decline and Fall... mixed with a healthy dose of Ludwig Boltzmann? Ben Bova's works are a socialist critique of modern capitalist excesses. It goes on and on: well respected sci-fi can always be reduced to core themes that are universal to the human condition. There's rarely been a well respected and well read work of science fiction which is just a series of fanciful inventions that goes nowhere interesting. They can be adventure stories, social critiques, religious allegory, whatever, but science fiction is merely a setting and not a theme unto itself, at least not when it is done by those that critically and commercially are regarded as doing it best. --Jayron32 22:17, 6 November 2012 (UTC)[reply]
RE: 'In good science fiction, the "science fiction" elements blend in or melt away in ways that make them part of the background of a really good story'. I disagree. I want the sci-fi elements up front. One thing I disliked about the original Outer Limits was how they often felt the need to put some type of "human interest" story up front, like whether two of the characters will rekindle an old romance. If I wanted to see that, I'd watch a soap opera or read a Danielle Steel book. StuRat (talk) 22:59, 6 November 2012 (UTC)[reply]
Again, De gustibus non est disputandum. --Jayron32 04:12, 7 November 2012 (UTC)[reply]
Indeed. So don't tell people it's better not to try. There are oddities like Star Trek Star Fleet Technical Manual that actually sell, (besides, to the purist sales don't matter in matters of art). I think the most memorable sci-fi must honor both science and art - I find stories such as 20,000 Leagues Under The Sea and Blowups Happen to be some of the greatest of the genre, indeed, well worth appreciating even a century after the real technology has actually been developed. Wnt (talk) 15:45, 7 November 2012 (UTC)[reply]
Also note that while Clarke used magic technology in one of his monoliths, I think it was actually one of the weakest parts of 2001:A Space Odyssey (the ape-man gradually learning how to tie a crude knot, with the afterthought that "his DNA was being changed") By comparison TMA-1 was a simple land mine, activated by solar power and sending the signal that ultimately blows up Jupiter. Accurate technical details like the gravitational slingshot, or the space elevator from The Fountains of Paradise, were far more important for the success of his stories in my opinion. Wnt (talk) 16:03, 7 November 2012 (UTC)[reply]
The problem with The Stand isn't that it's a horror fantasy story, or that it contains mystical, magical elements that drive the characters and events of the second half of the book. It's that there's a 'bait and switch'—in that the first part of the book comes across as straight science fiction, almost hard science fiction. Back when it was published in 1978, the experienced genre fiction reader would have been exposed to books like Stewart's Earth Abides (1949), Shute's On the Beach (1957), and of course Crichton's The Andromeda Strain (1969). The Stand starts off in that familiar vein, working with a horrifying but plausible scenario. A reader has to get an awfully long way in to The Stand before realizing that King has abandoned his interesting apocalyptic (and post-apocalyptic) story for some hokey bullshit about demons in Sin City. It feels like King started out writing a character-driven science fiction epic, and then either ran out of ideas or got afraid that his 'regular' audience would miss the supernatural stuff, and so decided to lay some of it on with a trowel. I generally like it when an author does something creative and genre-busting, but in the case of The Stand, King produced an awkward Frankennovel rather than a seamless blending of sci-fi thriller with fantasy horror. TenOfAllTrades(talk) 04:03, 7 November 2012 (UTC)[reply]
I have to generally agree with that analysis, but the bait-and-switch is only a problem in so far as it affects the dramatic impact of the story. If you are rabidly anti-supernatural, obviously you will be annoyed by the turn, perhaps enough to make you stop reading. But the plot itself remains strong enough to carry the book to a happy conclusion. If you want great apocalyptic without the supernatural, see Larry Niven's Footfall and Lucifer's Hammer, Heinlein's Friday, and Greg Bear's Forge of God. μηδείς (talk) 16:19, 7 November 2012 (UTC)[reply]
The science of time travel only appears briefly in the beginning of my proposed story. It's what the main character does in the past that matters. I just wanted opinions on the plausibility within the realm of sci-fi. Regarding the question of the Brownian motion, that is well beyond my knowledge of physics. I only have a layman's understanding of various aspects that I've learned from popular science books. The suggestion that his oscillation would create miniature explosions and implosions is interesting. Sending air in different temporal directions reminds me of a video that some physics students made a few years ago in which they tried to explain the science behind Hiro Nakamura's ability to travel through time. They stated that he would have to take air with him or he would suffocate during the time travel process. --Ghostexorcist (talk) 17:28, 7 November 2012 (UTC)[reply]

--Ghostexorcist (talk) 17:28, 7 November 2012 (UTC)[reply]

Go with it making a vacuum and causing a small thunderclap from where he's left, as well as an increase in body heat, dizziness, belching, nausea, farting, and minor symptoms of the bends at the new location. There's no guarantee that the air pressure at the target location would be equal to the place left from in any case. μηδείς (talk) 17:37, 7 November 2012 (UTC)[reply]
Yea, it might make sense to transport them in a space suit, so they can then gradually equalize the pressure before removing the helmet. This is the level of detail I'd very much like to see in sci-fi. StuRat (talk) 19:39, 7 November 2012 (UTC)[reply]
But the space suit is only going to help if he telewhatevers into a vacuum. Thet being an obvious risk, he'd be a fool not to wear one. Yet he will still have to deal with the bends if he teleports into an atmosphere, regardless of space suit. μηδείς (talk) 23:06, 7 November 2012 (UTC)[reply]
Why is that ? Are you assuming the air already present will merge with his body ? That would be nasty, yes. StuRat (talk) 23:14, 7 November 2012 (UTC)[reply]
I noticed no one has covered what I think would be a pretty significant effect of his "rapid phasing". If he is phasing at a frequency anywhere between 20Hz and 20kHz then there will be a constant buzz, hum or ringing at that frequency. He and everyone around him would hear it as he was phasing. 209.131.76.183 (talk) 16:18, 8 November 2012 (UTC)[reply]

Mixing Oort clouds

I came across this diagram showing that in approximately 25,000 years Alpha Centauri will make its closest approach to the Sun - approximately 3 light years. The diagram indicates that the Oort cloud is about 1.5 light years in radius (somewhat more than the Oort cloud article says). Assuming that Alpha Centauri also has an Oort cloud similar in size to the solar system's Oort cloud, it seems possible the two clouds might overlap to some extent. Is there likely to be any mixing of material between the two systems? Astronaut (talk) 18:53, 6 November 2012 (UTC)[reply]

Possibly, but the extreme low density of both means that gravitational interactions will be minimal, much less any actual collisions. However, the orbital periods of Oort cloud objects around the Sun are so slow, that even the slightest deflection could have noticeable impact after a few such rotations (which is many thousands of yeas). StuRat (talk) 19:17, 6 November 2012 (UTC)[reply]
The orbit of Proxima Centauri is not known or even confirmed, so guesses about an Oort cloud around the Alpha Centauri trinary are beyond speculative. μηδείς (talk) 19:45, 6 November 2012 (UTC)[reply]
By "not even confirmed" do you mean it's not known whether Proxima has positive or negative energy with respect to Alpha A/B, or what? —Tamfang (talk) 20:03, 6 November 2012 (UTC)[reply]
Sources I have read, including our own article, which was the most recent, say that it is not yet confirmed whether Proxima Centauri is gravitationally bound to Alpha Centauri A and B. I unreservedly, and wholeheartedly, recommend The Sparrow. μηδείς (talk) 21:35, 6 November 2012 (UTC)[reply]
How do you define over-overlapping, is it a synonym for superoverlapping or repeat overlapping, what is the difference between overlapping and over-overlapping? Plasmic Physics (talk) 20:47, 6 November 2012 (UTC)[reply]
None of these. It was a simple typo; and one I have just corrected. Astronaut (talk) 12:47, 7 November 2012 (UTC)[reply]
In that case, certainly - the amount of mixing is related to the time spent in overlap, the longer it remains in such a state, the more mixing should occur. That's all relative though, two factors (weak gravity, diffuse cloud density) impede the amount of mixing. Plasmic Physics (talk) 23:30, 7 November 2012 (UTC)[reply]

IQ/social group

Are there any serious studies regarding IQ by social group. Like IQ of Christians/Jews/Muslim, IQ of women/men. I know that such things would make many people ruffle their feathers, but it could be perfectly scientific. Comploose (talk) 19:06, 6 November 2012 (UTC)[reply]

Those aren't "social groups". And, you're right that it would be far too controversial. Scientists don't enjoy having their motives and research questioned, and much less do they death threats. StuRat (talk) 19:13, 6 November 2012 (UTC)[reply]
What are they then? Comploose (talk) 19:20, 6 November 2012 (UTC)[reply]
Religion and gender. Not sure if there's a good collective term for them. Under anti-discrimination laws they are sometimes called "protected groups" (along with race, national origin, and maybe sexual orientation). StuRat (talk) 19:33, 6 November 2012 (UTC)[reply]
Please refer to race and intelligence.A8875 (talk) 19:18, 6 November 2012 (UTC)[reply]
Well, most of such studies of race are pretty dubious. It's easier to define gender than race.
First of all, you're not talking about gender; you're talking about sex. But anyway, it isn't necessary to have a precise definition of race to do a statistical study on what variables correlate with it. If you can get all observers to agree most of the time on what category someone belongs to, that's enough to see whether other things are correlated. The weakest part of such studies is probably trying to figure out what exactly you're measuring, not whom to count. --Trovatore (talk) 19:46, 6 November 2012 (UTC)[reply]
Maybe. Maybe not. A small, but not insignificant number of people are intersex or transgender (of various types). And that's not even getting into whether to categorize the various kinds of homosexuals and bisexuals in there somehow. Any time you try to categorize people in pretty much any way, you're going to find it stickier going than you could have imagined at the outset. Matt Deres (talk) 19:51, 6 November 2012 (UTC)[reply]
But once again, it really doesn't matter. The existence of intersex folks makes it problematic to claim that sex is a completely well-defined category, but it poses almost no problems for studying what correlates with sex, because the percentage of intersex persons is very small indeed. --Trovatore (talk) 19:53, 6 November 2012 (UTC)[reply]
Read The Bell Curve, if you dare. μηδείς (talk) 19:52, 6 November 2012 (UTC)[reply]
In addition to race and intelligence (already mentioned), we also have sex and intelligence and religiosity and intelligence. Dragons flight (talk) 20:06, 6 November 2012 (UTC)[reply]
it's 2012. scientists are publishing correlations between single nucleotide polymorphisms and phenotypes, and tracing the pathways linking them; talking about linking something as vaguely defined (whose actual existence as a discrete thing is debatable) as IQ, to something as crudely defined as race (based mainly on the degree to which one's outer integument processes tryptophan into melanin) is basically phrenology. IQ is just a test result in search of an underlying quantifiable, and race, sex, etc. are just visual classifications based on widely accepted sociological norms. (doubt it? how do you classify for this question of IQ, people who have XY chromosomes and female physiology, or those with XX chromosomes and male physiology? Whatever you answer, you've implicitly dropped the "sex" concept, and are now talking about physiology or chromosomes.) Gzuckier (talk) 02:43, 9 November 2012 (UTC)[reply]
I know I'm repeating myself, but it doesn't matter how you classify them. Or rather, it matters only if the effect size is very small. For at least 99.9% of your test subjects, the assignment as to sex is going to be completely unambiguous, so the ambiguous cases just add a little noise. --Trovatore (talk) 04:39, 9 November 2012 (UTC)[reply]
yeah, but you don't know if your effect due to 'sex' is genetic, or just sociological. it's the ones who are split on those two factors hat provide contrast in the statistical sense and enable you to differentiate between the two factors, causally. if you only stick with the cases where the two correlate you won't be able to answer what is presumably the form of the question most people are interested in: are the differences in IQ between your various groupings genetic/inherited, or just something society beats into their heads and/or biases the tests to show? Gzuckier (talk) 18:49, 9 November 2012 (UTC)[reply]
Interpretation of a study, particularly assigning causality, is always fraught. But that's an entirely separate issue. Determining whether there is a correlation (the actual thing that statistics can help you with) is not made particularly more difficult by the absence of a precise definition of biological sex. --Trovatore (talk) 19:00, 9 November 2012 (UTC)[reply]


Fun chat about cats and dogs. But let's not forget the question is about human IQ. Wnt (talk) 23:34, 7 November 2012 (UTC)[reply]


Coming soon: Cat/dog preference and intelligence. Evanh2008 (talk|contribs) 21:07, 6 November 2012 (UTC)[reply]
Cats can live with their cerebra removed, dogs die. QED. Arrfff! μηδείς (talk) 21:31, 6 November 2012 (UTC)[reply]
these guys disagree [14] Gzuckier (talk) 02:59, 9 November 2012 (UTC)[reply]
It's clear that cats are smarter than dogs. A dog will do whatever you want him to do. How many intelligent people do you know who'll do that? --Trovatore (talk) 21:47, 6 November 2012 (UTC)[reply]
Dogs will do whatever you want them to do, if and only if you convince them that there is a reward in it for them. As will cats. The difference is, dogs are genetically predisposed to identify the possibility of a reward being provided by a "packmate" who is a stable fixture of the environment, whereas cats are genetically predisposed to identify the possibility of rewards suddenly popping up quasirandomly where they need to be pounced upon quickly, or else they get away and you can just forget about them. Gzuckier (talk) 18:55, 9 November 2012 (UTC)[reply]
It depends on whether you consider members of a political party to be "intelligent". ←Baseball Bugs What's up, Doc? carrots23:12, 6 November 2012 (UTC)[reply]
Ha! I always knew rebelliousness was a sign of intelligence. Thanks for that insight, Trovatore. -- Jack of Oz [Talk] 23:46, 6 November 2012 (UTC)[reply]
Cats aren't rebellious. (Neither are dogs, unless they've been mistreated, in which case they're usually just crazy.) The former are just too stupid in most cases to know what is wanted of them. Note that dogs are aware there' "master's" eye direction, while cats are oblivious. Might as well brag about your pet tape worm's independent streak. μηδείς (talk) 01:39, 7 November 2012 (UTC)[reply]
Mark Twain disagrees with you:
Couldn't have put it better myself. (Of course, I'm always listing the things I can put better than Mark Twain, but this isn't one of them.) --Trovatore (talk) 03:56, 7 November 2012 (UTC)[reply]
It's hard to figure why someone would think dogs are "smarter" because they're subservient whereas cats are "dumber" because they do what they want. ←Baseball Bugs What's up, Doc? carrots07:45, 7 November 2012 (UTC)[reply]
I was able to train my cat to beg for her dinner. Dogs are often initially trained by giving them snacks to perform, too, but later they will do the tricks without a food reward. Cats, on the other hand, refuse to do tricks without the reward. So who's smarter ? StuRat (talk) 09:14, 7 November 2012 (UTC)[reply]
Don't get me wrong. I love dogs too. The love and loyalty of a dog is a thing to behold. But the ugly truth known, I don't respect them as much as cats. --Trovatore (talk) 09:18, 7 November 2012 (UTC)[reply]

I have an aunt who lives in Atlantic City. She loves me unconditionally and always gives me gifts. Down the street there's a slot machine. It gives me money too, but only if I pay it first, and only if it wants too. It's obviously much more intelligent than my aunt, more independent, and more worthy of my respect. μηδείς (talk) 17:33, 7 November 2012 (UTC)[reply]

Medeis, I gather that you have never been friends with a cat. I think that's a pity. I would counsel you to be open to offers of such friendship the next time one comes your way. --Trovatore (talk) 22:29, 7 November 2012 (UTC)[reply]
I have had pet cats and dogs. The best raised cats I have known were by a girlfriend who treated them as surrogat children. There were quite responsive to her stimulae, and attempted to breastfeed from her by pumping her chest with their feet well into old age. But they still didn't have anywhere near the intelligence, spontaneity, sociability, or intelligence of the average dog. There are stupid dogs, and even stupider dog owners. But the smartest cats don't come anywhere near an average dog in intelligence. I love all animals except incorrigible pests, so the only thing I really hold against cats is that they smell like cat piss. μηδείς (talk) 23:02, 7 November 2012 (UTC)[reply]
Not in a healthy, neutered cat they don't. If they are urinating on themselves, something is seriously wrong. An intact adult male is liable to urinate anywhere, though. And a bladder infection can cause a cat to urinate in unusual places, too (they seem to associate the pain during urination with the location, causing them to urinate in a new spot each time).
I've found dogs to smell far worse, between vomiting up roadkill they eat and urinating/defecating wherever, when you don't walk them quite soon enough. They also seem to have worse body odor, unless you bathe them regularly. And, all things being equal, a dog which weighs 10 times as much ought to generate 10 times as much stink. For example, a cat fart is probably too little to notice, but this is not true of a St. Bernard. StuRat (talk) 23:18, 7 November 2012 (UTC)[reply]
Not enough free medical advice here for you, Wnt? :) μηδείς (talk) 02:09, 8 November 2012 (UTC)[reply]

Tipler Cylinder

The maths behind the Tipler cylinder seems to be very hard to find both on wikipedia and elsewhere. What exactly would happen to an object, say a rocket of some sort, that approaches it? The article says it can travel backwards in time, does this mean it exceeds the speed of light in some form? Wouldn't the rocket be affected by G-force and/or pressure and gravity?

I found out about the Tipler cylinder through a horizon documentary and it said it provided a possible way to travel back in time as oppose to a black hole, in which you cannot escape the event horizon. Does this mean that if a Tipler cylinder existed it would not have any form of event horizon? And that at any point the rocket could leave it's orbit? Does this mean light can escape it too? — Preceding unsigned comment added by 109.153.170.218 (talk) 21:59, 6 November 2012 (UTC)[reply]

Also some sources say it has to be infinitely long whereas others say it just has to be sufficiently long for the rocket to ever avoid the ends of the cylinder. Which are correct? — Preceding unsigned comment added by 109.153.170.218 (talk) 22:05, 6 November 2012 (UTC)[reply]

The spacetime contains closed timelike curves. The "timelike" part of this essentially means "slower than light"; you don't exceed the local speed of light but nevertheless end up back where you started, in space and time.
I see no reason to think that you couldn't arrange to make acceleration (gee forces) and tidal forces as small as you like on one of these closed timelike curves, but I'm just guessing.
Tipler says in the paper that there are closed timelike curves going through every pair of spacetime points, which means there are no event horizons: you can literally get anywhere in this spacetime from anywhere else.
Tipler argues in the paper that the cylinder doesn't need to be infinite, just very long, but he's just guessing. His arguments are (1) in Newtonian gravity the infinite cylinder solution is a good approximation to finite cylinders; (2) when the length goes to zero you get the Kerr black hole solution, which everyone believes in. However, the Kerr solution has an event horizon, so it's possible there's always an event horizon in the finite case large enough to prevent time travel. It's not clear why that would be true but it may well be true; Hawking conjectured that it is.
(For reference, here's the original paper and the metric in question is
where a is the angular velocity of the cylinder, R is its radius, aR > 1/2 (in units where G = c = 1) and
. This is a bizarre metric and doesn't look physically plausible to me. The coefficient L, in front of dφ², is sometimes negative, and since φ is cyclic (it's an angle), there are obvious CTCs in the form of loops around the cylinder at distances where L is negative. Presumably you can navigate to any spacetime location by spiraling around the cylinder at that distance as many times as needed.) -- BenRG (talk) 02:47, 7 November 2012 (UTC)[reply]

Thank you! That's exactly what I was looking for! You have been so helpful you don't even know! — Preceding unsigned comment added by 109.153.170.218 (talk) 18:02, 7 November 2012 (UTC)[reply]

Apparently the problem with the finite length cylinder is that, due to its extreme mass, it would be expected to collapse down to a disc in short order.[15] Wnt (talk) 18:34, 7 November 2012 (UTC)[reply]

Does a full moon cause higher tides, and if so, how much higher?

This was part of the story of Hurricane Sandy. Many tellers of the story included the "fact" that the full moon would make the flood water levels higher. Does it? How much? HiLo48 (talk) 22:53, 6 November 2012 (UTC)[reply]

Spring tide has the details. It's a well-known and well-studied phenomenon. AlexTiefling (talk) 22:58, 6 November 2012 (UTC)[reply]
As for "how much", I think it's generally in the 20% ballpark (with a corresponding reduction for neap tides), though I'm sure it varies by locale. You can look up tide gauge data for the US from NOAA; here's the Atlantic City, NJ gauge for the coming month. Tide range on the 15th is about 6.5 feet, contrasted with 3.5 feet on the 8th (still for the larger tide; the smaller tide that day is only 2.5 feet). — Lomn 23:26, 6 November 2012 (UTC)[reply]
(ec)Oh, and clarifying: the moon making the flood levels higher is, for all practical purposes, strictly a matter of the spring tide. The storm surge is over and above the normal tide level to yield the total flood height, but a hurricane producing an 8 foot surge at full moon will still produce an 8 foot surge at a half moon, too. Sandy was particularly bad in that landfall near NYC coincided with high tide and spring tide, though it fortunately didn't coincide with the higher high tide (see this plot from Sandy Hook, NJ right up to failure). — Lomn 23:33, 6 November 2012 (UTC)[reply]
It may be well known and well studied, but from my perspective, it's rarely well explained and often misunderstood. The only quantitative measure I can see in the Spring tide article is "causing tidal differences of inches at most". I have the impression that there's a popular misconception that because the full moon is brighter, the moon somehow causes much bigger tidal differences. The discussion at Talk:Hurricane Sandy#Should the Full Moon be mentioned? gives me just that impression. HiLo48 (talk) 23:30, 6 November 2012 (UTC)[reply]
Well, note that "inchest at most" is specifically referencing perigean spring tide particularly, and is "inches" with regards to an average spring tide. Spring tide vs average tide can be quite substantial. Another popular misconception (or just a factoid that's easy to miss) is that the full moon and the new moon produce identical extreme tidal effects. — Lomn 23:35, 6 November 2012 (UTC)[reply]
Well, I still don't see a clear answer to "How much?" Anybody? HiLo48 (talk) 04:17, 7 November 2012 (UTC)[reply]
I thought I was reasonably clear above. What else are you looking for? — Lomn 04:52, 7 November 2012 (UTC)[reply]
How much higher? There was a lot of qualification and negatives in your explanation. Is there a simple answer? Did you look at Talk:Hurricane Sandy#Should the Full Moon be mentioned? HiLo48 (talk) 07:05, 7 November 2012 (UTC)[reply]
The size of the tides varies greatly from one location to another. But one statistic that can be given precisely is in the "Forces" section of the article: the solar tidal force is 46% of the lunar. The spring tide occurs when these two occur at the same time. (The thing that can confuse the student is that although the color of the moon obviously doesn't matter, it indicates its position relative to the Earth and Sun and therefore whether the two tides occur at the same time)
I should add that I don't actually know that the highest tide must occur precisely at the full/new moon - I can picture a scenario where the solar tide raises the level of a long body of water and gives the lunar tide more water to work with when it hits a little later, but I don't know if that is physically relevant anywhere in the real world. Wnt (talk) 15:36, 7 November 2012 (UTC)[reply]
The spherical cow simple answer is "about 20%". For Atlantic City, NJ (selected for proximity to Sandy's landfall, and not being disabled like Sandy Hook), spring high tide is about 1 foot above mean high tide over a mean tidal range of about 5 feet. Atlantic City's neap high tide is similarly about 1 foot below mean high tide, and spring/neap low tides vary similarly versus mean low tide. Thus, I conclude that the cows in New Jersey are approximately spherical. — Lomn 23:41, 7 November 2012 (UTC)[reply]

November 7

Making eggs on an industrial scale

How do egg farmers get roosters to service the massive quantities of hens on an industrial scale? Or do hens make eggs without getting any sex that cannot become chicks? Or can chickens reproduce parthenogenetically? 67.163.109.173 (talk) 00:10, 7 November 2012 (UTC)[reply]

hens make eggs without getting any sex that cannot become chicks. Dauto (talk) 00:19, 7 November 2012 (UTC)[reply]
What are you trying to say Dauto + OP? I suppose you both meant: "hens lay eggs without sex." — Preceding unsigned comment added by OsmanRF34 (talkcontribs) 00:24, 7 November 2012 (UTC)[reply]
You just have to insert proper punctuation to make it make sense: "hens make eggs – without getting any sex – that cannot become chicks". Someguy1221 (talk) 00:51, 7 November 2012 (UTC)[reply]
OK, that sounds like a cautionary tale about how important punctuation is. There is still the problem of saying "make eggs" instead of "lay eggs", as if the chicken were preparing the eggs. OsmanRF34 (talk) 00:56, 7 November 2012 (UTC)[reply]
I guess you're on my mother's side of a minor dialect divide between us: it grates on me when she says "make a potato" to mean "cook a potato". —Tamfang (talk) 01:06, 7 November 2012 (UTC)[reply]
  • If the hen doesn't make the egg, where does it come from? Of course the hen makes the egg (whether you are used to the phrase or not). Laying (expelling) is just the final step, after forming/making the egg. I see no basis for "correcting" that phrase. -- Scray (talk) 04:17, 7 November 2012 (UTC)[reply]
I'd use parens: "Hens make eggs (without getting any sex) that cannot become chicks." Or, if we don't want such a clumsy sentence: "Unmated hens lay infertile eggs." StuRat (talk) 01:01, 7 November 2012 (UTC)[reply]
The hens will lay eggs without ever mating. Presumably they have been bred for enhancement of this trait. The vast majority of eggs you buy at the grocer's will be unfertilized and contain no embryo. At a low rate, unfertilized chicken eggs will begin development through parthenogenesis, but they almost always fail before hatching. We have little bit at Parthenogenesis#Birds. Someguy1221 (talk) 00:24, 7 November 2012 (UTC)[reply]
It's worth remembering that human females create eggs without having sex too. They're generally not much use though. (The eggs, not the females.) HiLo48 (talk) 00:43, 7 November 2012 (UTC)[reply]
Meh, the females aren't of much use either. Yes, but the intact egg never leaves a human female's body. It disintegrates in the fallopian tubes if it isn't fertilized within a day or so. --140.180.252.244 (talk) 01:59, 7 November 2012 (UTC)[reply]
Egg farmers who supply eggs for hatching are advised to include one rooster for every 7 to 15 hens (depending on breed, and age and health of the roosters) to ensure fertility. Hens store sperm for a week or more, so daily sex is not necessary to produce fertile eggs. If there are too many roosters, they can injure the hens, or sometimes they fight and kill each other. Dbfirs 18:23, 7 November 2012 (UTC)[reply]
Does that figure include that they want some actual chicks, to replace the hens and roosters as they die (or are eaten) ? StuRat (talk) 19:32, 7 November 2012 (UTC)[reply]
If I'm reading it right, that's the figure for farms that are mainly or wholly producing actual chicks, for sale to the egg-producing farms. AlexTiefling (talk) 12:43, 8 November 2012 (UTC)[reply]
Yes, though the eggs are often sold to hatcheries which then sell the males to be fattened for food and the females to farms that produce eggs for eating. The latter farms usually have no roosters (why would they bother to feed unproductive birds, and many consumers don't like to eat fertile eggs). Hens are often purchased at "point of lay" by commercial operations, though some small farms might raise their own birds. Dbfirs 13:51, 8 November 2012 (UTC)[reply]

USDA has a lot of information on poultry and egg production available here. Zoonoses (talk) 06:03, 8 November 2012 (UTC)[reply]

Whatever you do, don't let any renegade roosters mate with your eggs and fertilize them unless you want scrambled chicks with bacon. μηδείς (talk) 19:39, 8 November 2012 (UTC)[reply]
Mate a boar with your hens and have a complete breakfast in a shell. Gzuckier (talk) 18:57, 9 November 2012 (UTC)[reply]

The Sun

Why is the Sun's atmosphere layer so much hotter than the Sun's surface? Bonkers The Clown (talk) 07:16, 7 November 2012 (UTC)[reply]

See Corona#Coronal heating problem. Plasmic Physics (talk) 10:53, 7 November 2012 (UTC)[reply]
It's not the "atmosphere" as such which implies the ions nearest the "surface" of the sun, but rather the higher-up ions which are excited by and accelerated by the stellar magnetosphere. They are quite "hot" but not very dense. μηδείς (talk) 22:54, 7 November 2012 (UTC)[reply]
Also, for the record, the outermost layers of the Earth's atmosphere are also the hottest (see Thermosphere), though at those densities, temperature has little meaning. Though it has a nominal temperature of 1500-2500 C, you'd freeze to death there as there aren't enough molecules to transfer enough energy to you to keep you warm. --Jayron32 02:42, 8 November 2012 (UTC)[reply]
Is that really true, though? after all, your body generates heat, and if you were in an insulator (i.e. a vacuum) I'd guess (totally guesswork, admittedly) you'd have to heat up pretty good before your radiated energy balanced your metabolic output. being in a thin atmosphere of very hot gas wouldn't make it any cooler. Gzuckier (talk) 03:04, 9 November 2012 (UTC)[reply]
No, you radiate energy faster than you generate it (calculations here). The fact that you haven't dropped dead is mostly due to radiation returning to you from your surroundings (though to be fair the effects of convection and conduction are much more potent than radiation for room temperature objects, so you could probably do without the inbound radiation so long as you weren't in a vacuum). The insulative properties of your body/clothing have a relatively small effect, but it's difficult to estimate what it will be, especially since your skin will cool off much faster than the rest of your body when you get cold (we had a discussion about this a few months ago). Jayron's point is that the "thin hot atmosphere" isn't going to efficiently transfer heat to anything, or from anything. Someguy1221 (talk) 03:21, 9 November 2012 (UTC)[reply]
Someguy has what I was trying to say. In the vacuum of space, you emit more heat as radiation than a combination of absorption and internal heat sources can compensate for. The thermosphere, despite its nominal temperature, is close enough to a vacuum to have little effect on transferring heat to you're body via conduction. --Jayron32 06:17, 9 November 2012 (UTC)[reply]
Wow, I've had it wrong all these years. My fault for just assuming without doing the math, the exact thing I love to chide others for doing. So, those movies where the guy is ejected from the airlock without a spacesuit and freezes don't have it wrong after all. Thanks for straightening me out so I have fewer chances to make myself look dumb in public. Gzuckier (talk) 19:01, 9 November 2012 (UTC)[reply]

can you please explain to me what are the rules to followed in deriving the electron configuration — Preceding unsigned comment added by Yukuri (talkcontribs) 11:34, 7 November 2012 (UTC)[reply]

Why don't you try googling "electron configuration", or even just enter "electron configuration" in the Wikipedia search box. Assuming you mean electron configuration of atoms of course. For all we know, you could mean electron configuration of molecules, or even the electron cloud configuration in an electronic device. Then, come back to us if there is a particular aspect that you have difficulty with. We are more likey to spend some time providing you with good help if we can see you have put some work in. Wickwack 120.145.9.226 (talk) 15:32, 7 November 2012 (UTC)[reply]
I've linked all previous mention of electron configuration for the OP's benefit and convenience. DRosenbach (Talk | Contribs) 22:09, 7 November 2012 (UTC)[reply]
I could have asked why you encouraged the OP's laziness by providing direct links to WP articles. However, you actually provided links to the wrong article each time except for one, so actually you are potentially misleading the OP. Electron config in atoms is not the same as molecules, and is not the same as cloud configuration in electronic devices. Wickwack 121.221.230.208 (talk) 23:56, 7 November 2012 (UTC)[reply]
They all link to the same, and seeing how the OP asked about electron configuration, that's where I sent him. If he needs something else, he'll either figure it out from there or he won't -- I don't think that's anything to get upset about. But either way, I provided him with direction, which is what I figure we're all here to do. DRosenbach (Talk | Contribs) 04:07, 8 November 2012 (UTC)[reply]

What about sound in vacuum ?

We can hear the sound of a ringing bell on Earth but we cannot hear it in vacuum or on moon because sound needs medium through which it can travel. Suppose, if we bring a bell on moon and the bell is ringing, it will produce sound energy or not. If the bell is producing sound energy, what happen to this energy ? Suppose, if we hit moon's surface with something hard and a person touches the moon's surface with his ears, will he listen the sound produced in this case ? Here, moon's surface is material through which sound can travel (I think). Thanks, for answering my both questions. Sunny Singh (DAV) (talk) 12:20, 7 November 2012 (UTC)[reply]

You're correct - a person putting their ear to the moon's surface will hear the other person tapping. You can demonstrate this yourself as part of the famousalarm clock in a vacuum jar experiment - if the alarm clock is resting on a suitably hard table, you should still be able to hear it by putting your ear to the table.
Of course, I'm making a few assumptions in saying that someone would be able to hear tapping on the moon. First, that the listener could place their ear directly on the ground - that is, get their ear outside any helmet they might be wearing without suffocating or having their head explode. Secondly, that the distance between the tapper and the listener is not too great. Sound propagates through a surface in all directions, with the energy in any one direction being quite low. The amount of sound energy received by the listener will be inversely proportional to the distance from the source - possibly either by the inverse of the square of the distance, or even the inverse of the cube of the distance. (My maths has temporarily failed me on this point.) Thirdly, we assume that the moon's surface is of a suitable substance to allow decent sound propagation. If you think about it, sound travels better through a wooden table than through a cushion. The harder the moon's surface, presumably the better the sound will travel to the listener. - Cucumber Mike (talk) 12:46, 7 November 2012 (UTC)[reply]
Taking off the helmet shouldn't be necessary, if at least part of the helmet is rigid. Hm, have astronauts ever tried the old fictional trick of switching radios off and holding helmets in contact to talk privately? —Tamfang (talk) 17:11, 7 November 2012 (UTC)[reply]
(ec) :Your assumption about the moon's crust conducting the sound is essentially correct. If the bell is ringing at a substantial volume, a nearby astronaut might be able to hear it as the sound waves disperse across the moons surface and through the boots of his spacesuit. If, on the other hand, the bell is in pure vacuum and not in contact with any other body, the sound's energy will theoretically remain trapped within the bell itself indefinitely, though I suspect that the actual form of that energy (sound) would dissipate throughout the bell and equalize rather quickly, most likely taking the form of simple heat before too long. No energy would cease to exist over time or be sucked away to some phantom dimension; it all remains in the bell until the bell either impacts another body or ceases to exist. An actual waveform is not going to propagate indefinitely, however. Evanh2008 (talk|contribs) 12:47, 7 November 2012 (UTC)[reply]
The Apollo missions each left the Apollo Lunar Surface Experiments Package on the moon. These experiments included active and passive seismic equipment. Apollos 13, 14, 15, 16 and 17 deliberately crashed the third stage of the Saturn rocket into the Moon, and the seismic experiments detected the vibrations. So, yes the moon's surface does conduct sound. Astronaut (talk) 16:13, 7 November 2012 (UTC)[reply]
The Moon seems to be covered with a thin layer of dust, which would not propagate sound well. You'd need to make contact with the rock below, both with something connected to the bell, and something connected to the astronaut's helmet. StuRat (talk) 19:27, 7 November 2012 (UTC)[reply]
As for the more interesting question of a bell rung in a pure vacuum, I imagine it would continue to vibrate for a very long time. (The vibrations would eventually all be converted to heat, but that's much slower.) StuRat (talk) 19:29, 7 November 2012 (UTC)[reply]

Hover

How do you calculate the energy needed (by an helicopter or similar aircraft) to hover? Comploose (talk) 14:07, 7 November 2012 (UTC)[reply]

It's not really a matter of energy, because a stationary helicopter does not do any physical work. A stationary aircraft needs to create an upwards force that makes up for the gravitational force pulling down the aircraft, but it depends on the method of generating this force how much power/energy is needed for this. - Lindert (talk) 14:18, 7 November 2012 (UTC)[reply]
Lindert is completely wrong here with his inane answer. The helicopter is doing some work on the rotor, and obviously needs energy for that, which can be calculated. See the physical details [| here]. OsmanRF34 (talk) 14:33, 7 November 2012 (UTC)[reply]
If you bothered to read the link you provided, you will noticed that the person who calculated the necessary power came to the conclusion (correctly) that dE/dt = 0.5*M*g*v, where v is the velocity of the air that is being pushed down. As you can see, the energy is proportional to the downwards velocity of the air, or in other words, the slower the air is being pushed down, the less energy it will cost. Of course you need to compensate for the slow air by increasing the airflow, but essentially this means that if you increase the airflow to infinity, the energy will go to zero, so theoretically, no energy is needed, though this is not implementable in practice. The point of my post above, as you might have noticed, is not that you need no energy, but that the required energy is not a fixed amount, but rather depends on the kind of aircraft. - Lindert (talk) 14:51, 7 November 2012 (UTC)[reply]

It's not an insane answer. Imagine a large feather that falls in a still church and lands halfway off of a chair. Now we seal the church for millennia. First, what is keeping the feather from falling to the ground? It's the chair, doing work. So, when does it run out of work that it can do? After 1000 years? 10,000? A million? It never runs out, because it doesn't perform work. Theoretically, the chair could suspend the feather forever.

Likewise, if you are trying not to fall by suspending yourself in air, all you have to do is push down against something in the same way that the chair is connected to the ground - except instead of chair, you have air. Theoretically, there is no work that actually needs to be done. On a practical level, air will move out from under you whereas chair does not. But that is an implementation detail. --91.120.48.242 (talk) 15:12, 7 November 2012 (UTC)[reply]

And if Lindert had read the webpage Osman provided, he would have seen that it is for the case of a bird hovering well above the ground, ie high enough that any ground effect can be neglected, similar to a helicopter. This is not the case for a hovercraft. A hovercraft only rises enough to let air out around the skirt. In this case the viscosity of the air casues resistance to the flow of air out between the ground and skirt, such that the air pressure within the skirt is just sufficient to support the weight of the craft. The energy (more correctly the power) required is that which is lost in the visocity friction around the skirt. If the engine/fan supplies less than that, the hovercraft will sit on the ground. If more than the required power is supplied, the skirt-to-ground gap will increase unnecesarily, allowing the air velocity to increase to maintain the same pressure, and the viscosity friction loss must increase to match the increase in engine/fan power. The energy consumed by a hovercraft is thus a minimum at the minimum skirt-to-ground clearance, and increases with increasing clearance, until the machine is operating as a helcopter. Wickwack 120.145.9.226 (talk) 15:22, 7 November 2012 (UTC)[reply]
There is no contradiction. The helicopter is not doing work by staying up - it is doing work by circulating air (and making noise, and heating the air, etc.). To illustrate, suppose you build a "helicopter" out of thin, horizontal plastic wrap that is the size of the Earth, completely surrounding the entire atmosphere at a certain level. It takes no circulation at all to hold up. Wnt (talk) 15:28, 7 November 2012 (UTC)[reply]
I'm sorry, but I see nowhere in the question that this is about a hovercraft. The OP specifically asked about a helicopter or similar aircraft, so this is perfectly analogous to a bird in the sense that it flies high above ground. - Lindert (talk) 15:33, 7 November 2012 (UTC)[reply]
You are right - I was off in fairy land for some reason. My appologies. You are correct. Wickwack 120.145.9.226 (talk) 15:38, 7 November 2012 (UTC)[reply]
No problem, I could easily see myself making the same mistake. - Lindert (talk) 15:42, 7 November 2012 (UTC)[reply]
(ec) Lindert, Wickwack, and 91.120 are spending a lot of time giving the right answer to the wrong question. Their calculation answers the question "how much work is being performed on a hovering helicopter?", which does indeed come out to zero. What the OP asked for, however, is not how much work is done (or how much power is required) to hold a theoretical mass stationary in the air, but rather how much energy it takes to keep a real helicopter in the air. That answer is greater than zero.
What would be useful here would be (one or both of)
  • a method of approximately calculating the amount of lift generated by a helicopter rotor as a function of input power (there's probably a rough equation floating around somewhere that takes the rotor blade length and angular speed, the number of blades, the density of air, and some kind of blade shape fudge factor as parameters); and/or
  • some actual numbers for some real aircraft. Numbers surely exist for the actual, measured power output of helicopter engines under hovering conditions, and they will give you some idea of how (un)reliable estimates based on an approximation formula might be.
Come on, people—we're the Reference Desk. Let's find some references. TenOfAllTrades(talk) 15:44, 7 November 2012 (UTC)[reply]
Not to be rude, but how do you manage to read the mind of the OP to see what he meant? I certainly understood it in a theoretical sense: 'how much energy is needed', as opposed to 'how much energy is actually used'. Nonetheless, I appreciate your effort to answer the question. - Lindert (talk) 15:53, 7 November 2012 (UTC)[reply]
Thanks for Osman + TenOfAllTrades for you answers. That's exactly what I wanted to know. Comploose (talk) 16:26, 7 November 2012 (UTC)[reply]
(ec)The real-world answer is that some kind of helicopter I found on Google uses 45 gallons per hour of what I'll assume is some kind of Aviation fuel. [16] The article says BP Avgas (I don't know that's the right type) contains 44.65 MJ/kg with a density of 690 kg/m3 = 0.69 kg/liter, i.e. 30.808 MJ/liter - assuming the gallons are US liquid that's multiplied by 3.79 liter/gal to get 116.762 MJ/h = 32400 watts. (Although this is the power of only 22 hair dryers on high [17] apparently most of a hair dryer's energy goes into heat, so don't launch your lawn chairs until we work out the details) Wnt (talk) 16:29, 7 November 2012 (UTC)[reply]
Around here, they fly R22 and Robinson R44 helicopters and fill them with 100LL AvGas. As far as a reference: start with the Pilot's Handbook of Aeronautical Knowledge - which provides just enough balance between theoretical information and practical real-world knowledge. The FAA also publishes an airplane flying handbook, but for helicopters, all I found is the Helicopter Instructor's handbook, which covers physics of flight in Chapter 3.
From first principles of mechanics or thermodynamics, you won't get very far. Helicopter flight is characterized by aerodynamic work against a non-conservative force. What this means in practice is that the amount of energy or power required for a maneuver depends on the helicopter's configuration. A helicopter pilot can independently control the collective, the cyclic, and also throttle the engine rpm. The pilot can also change the attitude of the helicopter to maintain a hover at many different engine settings and aircraft attitudes. Phrased another way, the angle of attack of the rotors can be varied to change the aerodynamic efficiency of the helicopter, while maintaining a level hover or a stable hover at an unusual attitude. The blade pitch changes how efficient or draggy the blade is - and this directly affects how much lift is produced for a given amount of energy So, the total amount of energy the engine has to produce is going to vary signficantly. You can pull up the pilot's manual for an R22, or other helicopter, to see normal operating engine and control configurations for a hover. I see the engine maxes out at just around 2600 RPM, which will hover the aircraft in certain conditions, and lift it in other conditions; of course, hover performance is affected by weight and atmospheric conditions. The engine is rated for around 150 hp, but is designed to take off at around 120 hp, and can lift the aircraft at much lower power settings. The yellow line starts at 60% on the tach, and you can convert that to produced horsepower using the charts in the aircraft manual. Nimur (talk) 21:46, 8 November 2012 (UTC)[reply]

G-Force

If you accelerate in a vacuum, would you experience G-Force? — Preceding unsigned comment added by 109.153.170.218 (talk) 18:16, 7 November 2012 (UTC)[reply]

You would feel a force, and you could measure that force scaled by units of g, the standard measure of gravitational acceleration at Earth's surface. I personally don't like the term "G force" because it conflates the units with the effect; and I have seen it used in various contexts to mean totally different things. Nimur (talk) 18:42, 7 November 2012 (UTC)[reply]

Is there any way to prevent G-force/any circumstance you could travel at very fast speeds without being affected by g-force at all? — Preceding unsigned comment added by 109.153.170.218 (talk) 19:07, 7 November 2012 (UTC)[reply]

Yes, by having a large mass in front of you, which accelerates with you. It's gravitational attraction will thus counter the g's due to acceleration. The denser and closer the mass is to you, the less mass is required. Of course, there's no practical way to do this, at present and in the foreseeable future, as the mass would be huge. StuRat (talk) 19:18, 7 November 2012 (UTC)[reply]
I don't think you've worked that scenario out to its logical conclusion, StuRat. Nimur (talk) 19:30, 7 November 2012 (UTC)[reply]
Meaning that they would have to rapidly accelerate a mass on the order of the size of a planet ? Yes I have. That why it's not practical in the forseeable future. Perhaps with a mini-black hole, the mass could be far less, but then you have the issue of how to create the mini-black hole, keep it stable, and keep it from swallowing the ship, while accelerating the whole thing. My much more modest design involves a massive linear particle accelerator which clamps on to a metallic asteroid. The asteroid is kept in front, and acts as a shield, while it is mined and shot out of the linear accelerator at the speed of light. If the asteroid generates a 0.5g gravitational attraction, then a 1.5g acceleration would only be felt as 1.0g, to those on the surface. StuRat (talk) 19:55, 7 November 2012 (UTC)[reply]
Stu, I wanted to say that I think your planet-idea is brilliant, literally brilliant. No one has included this in the inertial-dampener discussion. 91.120.48.242 (talk) 13:03, 8 November 2012 (UTC)[reply]
that's basically how we are travelling through space right now, isn't it? just like the idea that you can buy a fusion powered boat which keeps the reactor a safe 93 million miles away and transfers energy by radiating it into a working fluid of ordinary atmospheric air, which is captured by a simple structure of large fabric vanes.... Gzuckier (talk) 03:10, 9 November 2012 (UTC)[reply]
There's a short story by Charles Sheffield about a ship whose nose is a big ball of neutronium. The passenger pod moves up and down a mast, adjusting its distance from the neutronium so that its gravity balances the acceleration of the engine (whose magic unlimited energy source I do not remember!) to within ±1g. At maximum acceleration, a ratchet malfunctions so the pod gets stuck close to the neutronium, so the ship can't slow down ... —Tamfang (talk) 22:27, 7 November 2012 (UTC)[reply]
This story, but as I'm not strongly motivated to re-read it I don't guarantee that my summary is accurate in detail. —Tamfang (talk) 22:31, 7 November 2012 (UTC)[reply]
Also, you said "fast speeds", but I assumed you meant "fast accelerations". There's no problem going at fast speeds, as the g's are only caused by the acceleration. Thus, if you accelerate slowly enough, you can go any speed you want, up to those approaching the speed of light, without excessive g's. Accelerating at 1 g (normal gravity), for example, you can get close to the speed of light in about a year. StuRat (talk) 19:21, 7 November 2012 (UTC)[reply]

Thank you! Whats the maximum acceleration a human can withstand? — Preceding unsigned comment added by 109.153.170.218 (talk) 19:49, 7 November 2012 (UTC)[reply]

That greatly depends on the length of time and what precautions are taken, and if they need to remain awake and able to act or merely stay alive. For seconds in a pressure suit, a properly trained and evaluated test pilot might be able to take on the order of 10 g's, while an ordinary person might only be able to take 1.1 g, for months at a time. StuRat (talk) 19:59, 7 November 2012 (UTC)[reply]
Colonel John Stapp pioneered the use of rocket powered acceleration/deceleration sleds in the late 1940's to test seat restraints for aviation, and repeatedly withstood accelerations of 45G, though he broke a wrist and suffered retina detachment. Edison (talk) 01:06, 8 November 2012 (UTC)[reply]

Strong magnetic fields can be used to counter the effects of the G-force, see e.g. here.

"Whether an object will or will not levitate in a magnetic field B is defined by the balance between the magnetic force F = M∇B and gravity mg = ρV g where ρ is the material density, V is the volume and g = 9.8m/s2. The magnetic moment M = (χ/ µ0)VB so that F = (χ/µ0)BV∇B = (χ/2µ0)V∇B2. Therefore, the vertical field gradient ∇B^2 required for levitation has to be larger than 2µ0 ρg/χ. Molecular susceptibilities χ are typically 10^(-5) for diamagnetics and 10^(-3) for paramagnetic materials and, since ρ is most often a few g/cm^3, their magnetic levitation requires field gradients ~1000 and 10 T^2/m, respectively. Taking l = 10cm as a typical size of high-field magnets and ∇B^2 ~ B^2/l as an estimate, we find that fields of the order of 1 and 10T are sufficient to cause levitation of para- and diamagnetics. This result should not come as a surprise because, as we know, magnetic fields of less than 0.1T can levitate a superconductor (χ= -1) and, from the formulas above, the magnetic force increases as B^2."

But would that be useful for humans ? I tend to think that a strong enough magnetic field to exert a significant force on a person would cause medical problems, like preventing blood from circulating by exerting force on the iron in the hemoglobin. StuRat (talk) 20:22, 7 November 2012 (UTC)[reply]
Perhaps for short time intervals these effects are not a big problem. You can e.g. think of a magnetic air bag in a car that will produce a very strong magnetic field for a short time. You then also have to consider if you can tolerate the huge emfs generated by that. Count Iblis (talk) 20:38, 7 November 2012 (UTC)[reply]
Our own magnetic levitation article describes the levitation of a frog with an electromagnet. The frog seemed totally fine in the short term. I'm not sure if they ever did a long-term viability assay. Someguy1221 (talk) 01:29, 8 November 2012 (UTC)[reply]
And that's only 1 g on a small organism. I'd suspect that smaller organisms would be better able to handle it. StuRat (talk) 01:48, 8 November 2012 (UTC)[reply]

You could put the human inside a liquid-filled tank and also fill all air-containing spaces inside the human body with the liquid (see liquid breathing). The density of the liquid should be similar to the density of the human body, which prevents the lungs and other cavities from collapsing. The acceleration limit will be higher, but due to different tissues having different densities, there will still be a limit. With the bone density of maybe 1.5 g/cm3, the adipose tissue density of maybe 0.9 g/cm3 and the density of air of approximately 0, one would think that the tolerable acceleration is multiplied by a factor of (1.5 - 0)/(1.5-0.9). But that's of course a simplification, as a bone usually isn't directly surrounded by adipose tissue (and for what's inside the bone, you have to take into account that the bone is mechanically more stable than the lung). How would death or injury occur at high accelerations, given this liquid immersion? Icek (talk) 20:51, 7 November 2012 (UTC)[reply]

That gimmick was used in The Forever War, right? —Tamfang (talk) 22:32, 7 November 2012 (UTC)[reply]

Okay, Thank you! — Preceding unsigned comment added by 109.153.170.218 (talk) 20:59, 7 November 2012 (UTC)[reply]

Sorry I was a few comments behind there! Clicked edit, didn't see the new comments! This is all extremely interesting and useful! Do you know if anyone has already worked out the calculations on liquid breathing affecting the acceleration? Or are there any similar workings that could come in useful? — Preceding unsigned comment added by 109.153.170.218 (talk) 21:06, 7 November 2012 (UTC)[reply]


Stu, I wanted to say that I think your planet-idea is brilliant, literally brilliant. No one has included this in the inertial-dampener discussion. 91.120.48.242 (talk) 13:03, 8 November 2012 (UTC)[reply]

Thanks. You're quite welcome to use it. Note that such a ship would have to find and clamp onto a new asteroid, when it arrives at it's destination, to be used for the return trip. In other words, it would use multiple asteroids in Ceres. :-) StuRat (talk) 18:48, 8 November 2012 (UTC)[reply]

Generating energy without losing fuel

I am not sure about this but this question is revolving around my mind for several days. Suppose, in nuclear reactor, we are able to hit a proton with a neutron; proton will decay in neutron + positron + electron neutrino and neutron will decay in proton + electron + electron anti-neutrino. In reactor following may take place-
Proton + neutron → neutron + positron + electron neutrino + proton + electron + electron anti-neutrino.
In the above equation, positron is the antiparticle of electron and electron anti-neutrino is the antiparticle of electron neutrino. Annihilation will take place between particles and their corresponding antiparticles with the release of huge amount of energy. The above equation can now be restated as-
Proton + neutron → neutron + proton + energy.
Hence, energy is produced without losing fuels (i.e. proton and neutron). I am not sure whether this process is right or wrong. Is it possible to generate energy in this way ? Thank you! Sunny Singh (DAV) (talk) 18:58, 7 November 2012 (UTC)[reply]

What you are proposing is called perpetuum mobile. :) Ruslik_Zero 19:00, 7 November 2012 (UTC)[reply]
Won't the neutron decay? So you'd effectively lose a neutron in the equation? --Wirbelwind(ヴィルヴェルヴィント) 19:04, 7 November 2012 (UTC)[reply]
What you're missing is that the two component reactions you're adding together don't happen with ground-state protons/neutrons (at least the one starting with the proton doesn't). So the net reaction is really energetic proton + neutron → neutron + proton + energy, which shouldn't surprise anyone. If you carefully examine the conditions where each half reaction holds, you'll find mass-energy is conserved in each, so even if you were able to combine them somehow, mass-energy would still be conserved. The energy you get out is exactly the amount of energy you need to pump into the proton/neutron to get the reaction to start. -- 205.175.124.30 (talk) 19:34, 7 November 2012 (UTC)[reply]

Who created this picture of the thermoscope?

In this blog, there is a picture of a thermoscope that is taken from an unknown book (to me). I have no idiea on this book. Do you know who created it or in which origin book it's found? Thank you. מוטיבציה (talk) 20:21, 7 November 2012 (UTC)[reply]

This site attributes it to the 1832 Edinburgh Encyclopædia. Regards, TransporterMan (TALK) 20:41, 7 November 2012 (UTC)[reply]
Thank you a lot, for the help. מוטיבציה (talk) 04:33, 8 November 2012 (UTC)[reply]

November 8

Melatonin

I am unclear in reading the article on Melatonin about its chemical properties:

  • Is the carbon ring structure an aromatic/benzene ring, or just alternating single/double bonds?
  • Is there a name for the structure of the carbon rings, the one similar to purine but not a purine?
  • Is melatonin technically an ether?
  • Is melatonin technically a ketone?

75.73.226.36 (talk) 02:25, 8 November 2012 (UTC)[reply]

For reference, this is Melatonin
The ring is aromatic, via Hückel's rule any ring is aromatic if it is planar and has 4n+2 electrons which can participate in pi-bonding. That fused 6-and-5 member rings with the nitrogen on the 5 membered ring has 10 pi-electrons (8 via double bonds, and 2 from the lone pair on nitrogen), which meets Hückel's rule (4*2+2 = 10). That ring structure is called Indole. There is an ether (the methoxy group attached to the indole) but there isn't a ketone group. The carbonyl attached to the amine is actually called an amide. --Jayron32 02:35, 8 November 2012 (UTC)[reply]
OK, I should have known about the amide factor. And I'm used to seeing aromatic rings as a circle within a hexagon. Is there any distinctive properties of melatonin that it gets from having the amide and ether groups? I am aware that the indole formation comes from the fact that it derives from the neurotransmitter serotonin. What makes it different? 75.73.226.36 (talk) 02:42, 8 November 2012 (UTC)[reply]
Have a look at the Melatonin receptor agonist article. The "Structure-activity relationship (SAR)" section talks about the effect of various structural changes, which tells you about the possible roles of the parts of the structure, and the "Binding and pharmacophore" section talks in more detail about those roles. DMacks (talk) 02:49, 8 November 2012 (UTC)[reply]
Ramelteon, a melatonin receptor agonist
More questions this raises! I see how ramelteon acts to promote melatonin reception, slightly, but now I have run into a

new mystery. What is the term for the dotted-line bond between the tricyclic carbon formation and the rest of the compound? I know some stuff about organic chemistry but this is new and unclear to me. Is there any specific factors this bond plays in ramelteon? 75.73.226.36 (talk) 03:08, 8 November 2012 (UTC)[reply]

See Skeletal structure#Stereochemistry for what this notation means. I have no idea what biochemical effect this specific stereochemistry plays, but given the synthesis of the drug, "it's almost certainly important" because the route appears to go to special effort to pick this particular one. DMacks (talk) 03:14, 8 November 2012 (UTC)[reply]
Thank you much! However, why don't more molecules include stereochemical markings in their structural diagrams? You can't tell me that all other molecules are two-dimensional. Are these connected to stereoisomers; that is, would solely a change in the direction of the stereo-bond change the chemical identity of the substance? 75.73.226.36 (talk) 03:22, 8 November 2012 (UTC)[reply]
In biologically active molecules, stereochemistry always matters. Levomethamphetamine is a nasal decongestant. Dextromethamphetamine is speed. The difference is the stereochemistry around the one stereogenic carbon. In melatonin, there are no stereogenic carbons, so you don't see any dashes or wedges. In Ramelteon there is a stereogenic carbon, and it matters, so it is shown as a dash (away from the viewer). --Jayron32 05:59, 8 November 2012 (UTC)[reply]
The markings for stereochemistry are only shown when they're relevant. If you look at the article on isobutane, the very simple formula at top is all that is used by normal standards. The one at bottom with all the hydrogens depicted is more for teaching, and even that doesn't bother to show which hydrogens are pointed toward you or away. That's because all those bonds can move - the molecule is achiral. It takes specific circumstances for the conformation to matter, such as a double bond that doesn't freely rotate, or the existence of a chiral center with four different ligands that can't be mirror-imaged onto itself. Wnt (talk) 16:47, 8 November 2012 (UTC)[reply]

Proposed Essay on zealotry and non-science

I would like to post an essay on this with reference to global warming and passive smoking. It there someone who could read this essay, advise if the subject has been covered before etc? Thanks in advance SmokeyTheCat 12:56, 8 November 2012 (UTC)[reply]

Please read Wikipedia:What Wikipedia is not#Wikipedia is not a publisher of original thought, especially item #3, as well as Wikipedia:No original research#Synthesis of published material that advances a position. Your proposed article sounds like it would run afoul of either of these long-standing and strongly enforced Wikipedia content policies. --Jayron32 14:43, 8 November 2012 (UTC)[reply]
Hmmm... it sounds like he's asking for a list of possible reviewers. Now to begin with, if the essay is descriptive of sources rather than personal opinion, and if you're willing to release it for copying and modification by anyone, it might be Wikipedia material, (or at least Wikiversity material. :) ) If not, we have a List of internet forums that is much less than satisfactory - it doesn't mention http://www.physicsforums.com for example (though I doubt that's quite what you want). Somebody help me out here - there must be a good resource listing lots of options for serious writers of pieces like this. Wnt (talk) 17:27, 8 November 2012 (UTC)[reply]
If you publish it on your blog, and announce it in a suitable place, I trust you'll get at least a few comments. —Tamfang (talk) 19:55, 8 November 2012 (UTC)[reply]
Thanks fellow editors. Does anyone ever read blogs? Three or four thoughtful, open-minded readers I all I seek.SmokeyTheCat 05:59, 9 November 2012 (UTC)[reply]

Actually this http://www.numberwatch.co.uk/religion.htm says most what I was proposing to say about global warming. SmokeyTheCat 06:04, 9 November 2012 (UTC)[reply]

Law of Conservation of Energy

I suppose such things would be improbably constructed in real life, but couldn't water be pumped to height X and then allowed to travel on an exceedingly long track of slide so as to trigger/interact with so very many water wheels etc. that more power is generated than it took to raise the water to the height X? Or is the universe constructed in a way such that, by definition, the conditions of the track of slide I propose (i.e. its relative lack of slope, etc.) would counteract my proposal by causing the water to proceed so slowly that little energy is produced because the wheels barely move. I'm not a formula guy, but is it this sort of logical manner of understanding the problem a good reason for why this won't work? DRosenbach (Talk | Contribs) 13:16, 8 November 2012 (UTC)[reply]

Yes, each wheel slows down the water so that the net effect is that significantly less energy is generated than the amount used to pump up the water. The only way you can get "more than 100% back" is to pump the water up at high tide and release it at low tide, though the "extra energy" in this case ultimately comes from the combined kinetic and potential energy of the earth-moon system. Dbfirs 13:40, 8 November 2012 (UTC)[reply]
(ec) I think you answered your own question quite well. The mechanical work that is done by a force on an object depends only on the distance that the object travels in the direction of the force. So because gravity pulls objects downwards, only the vertical component of the distance traveled can be converted to mechanical energy. Whether gravity works directly on the water, giving it speed, or whether the water transfers that work to e.g. a wheel does not matter here. - Lindert (talk) 13:45, 8 November 2012 (UTC)[reply]
Eventually, the water will be slowed so much by interacting with each wheel that it cannot turn the lower wheels at all, and starts to spill over the edge of its trough. AlexTiefling (talk) 14:43, 8 November 2012 (UTC)[reply]
And if you're hoping that each wheel is so small that its effect on the flow is negligible, that is unfortunately not the case. Suppose you make each wheel as small as you like. The amount of effect it has on the flow is subsequently "very small" - and so is the energy it can collect. Now, add up each miniscule contribution for each of your zillions of tiny waterwheels... and we've stumbled on the limit definition of the integral, without using any squiggly symbols. The mathematically-inclined observer will quickly make several critical observations: even very small quantities have a magnitude, and that magnitude can be compared to other very small quantities; and we've stumbled onto an epsilon-delta proof that no matter how small you make your water wheel paddles, they will each lose more energy than they gain. When you add up a large number of these water paddles, your total system cannot get more energy than its input. If you like to phrase it in this way, these mathematical facts correspond to physical observables; but the mathematics follow directly from our most fundamental axioms: in other words, most of us believe the universe is "constructed" in such a way that these facts cannot be incorrect (even if it takes a good deal of mental wrangling to deduce the consequences from first principles).
Incidentally, this is the argument I use to refute the apparent thermodynamic conundrum of Maxwell's demon. No matter how small you make the demon, his gate has to be larger than the atoms he wishes to trap. (Put another way, his potential energy barrier must be larger than the energy of the molecules). Therefore, each time he opens or closes the door, the demon must expend more energy than he recuperates using his "paradoxical" thermodynamic trap. The laws of thermodynamics are circumvented when you conveniently forget to count the lossy parts of the system - no matter how small the individual losses, they add up! Feynman elaborates on this model by endowing the door with Brownian motion, once again solidly demonstrating that the rules of thermodynamics - including conservation of energy in a closed system - are consequences of statistical mechanics and cannot be beaten. Nimur (talk) 15:27, 8 November 2012 (UTC)[reply]

Jeuming tree

Probably not it, but it would be cool if it were. Wnt (talk) 16:07, 8 November 2012 (UTC)[reply]

In a poem by China's 8th-century poet, Du Fu (or Tu Fu), translated into English as "Autumn Rains Lament 1," the subject of the poem is a man trying to save his 'jeuming tree' from being uprooted by winds. In various translations of the poem, 'jeuming tree' is the agreed-upon translated spelling. Yet, I can't find it either in my desk dictionary or in Wikipedia, "Flora of China."

Would you kindly locate and provide me with an online source for this tree?Speer46 (talk) 14:41, 8 November 2012 (UTC)[reply]

Thank you, Stanton Hager

This is a hard one to answer properly.
阶下决明颜色鲜 jiē xià jué míng yán sè xiān
著叶满枝翠羽盖 zhù yè mǎn zhī cuì yǔ gài
开花无数黄金钱 kāi huā wú shù huáng jīn qián
Below the steps, the jueming's colour is fresh.
Full green leaves cover the stems like feathers,
And countless flowers bloom like golden coins.[18]
is related to the genus Cassia (as confirmed by our zh.wikipedia entry [19] ) However, these are ill-defined groupings of plants. I am immensely tempted to point at the Golden Shower Tree (Cassia fistula, above) which is widely distributed and loved for its beauty [20], and sometimes flowers in fall [21] - however, the general location of Xi'an is at best marginal for growing it. (Then again, that might be the point) However, it is completely irresponsible for me to do so since it could really be one of hundreds of species of Cassia, Cinnamomum, or Senna. Wnt (talk) 16:07, 8 November 2012 (UTC)[reply]
Not sure if this is the same, but List of kampo herbs identifies 決明子 (jué míng zǐ)as Senna obtusifolia. That article also quotes the same poem. (Disclaimer: I have zero knowledge of Chinese). - Lindert (talk) 16:25, 8 November 2012 (UTC)[reply]
The wikt:子 is just "seed", the part used for that application; it's one of the many forms of "cassia" (maybe the same as Cassia tora, Senna tora, etcetera, maybe not). But looking at this I just don't feel poetic. Wnt (talk) 16:38, 8 November 2012 (UTC)[reply]
I'm not sure if this is any help. It apparently quotes the poem above and has some nice pictures of flowers - a bit more like "golden coins" than the one shown above. Alansplodge (talk) 17:28, 8 November 2012 (UTC)[reply]
Hmmm... some things mentioned:

Hmmm.... I'm seeing more shrubs and weeds here than trees. What I don't see is an answer about what this tree actually was. Indeed, the bottom line is that unless he mentioned it somewhere else, all we have is a poem about "cassia", and the precise nature of that is unobtainable. In the lack of a way to reason it out with any confidence, I'm going to stick to my perhaps poetic notion of it being a heroic effort to grow a Golden Shower Tree in Xi'an, even though the average low temperature in Xi'an is 21 F [28] and the seeds are only sold rated for Hardiness Zone 9b which has a minimum of 25 F minimum (i.e. central Florida) [29] or maybe even 10a [30]. To me the beauty of the plant and the difficulty of keeping one would seem to justify the poem. Wnt (talk) 19:02, 8 November 2012 (UTC)[reply]

Please help a new owner of a United Motors Matrix 150 motor-scooter?

Please don't post the same question on multiple desks. Any further responses will be added to the section on the Miscellaneous page
The following discussion has been closed. Please do not modify it.
Hello. I just bought this motor-scooter from a private seller on Craigslist for $700. The model-year was 2006, and has almost 2700 miles. (The scooter allegedly cost $2400 brand-new.) I am allegedly the 4th owner now.

After test-driving it for some time, I noticed that the braking power has weakened. I tried to send it to a Motorcycle Supply establishment in town, but the lead mechanic took a look and said that it’s an “oddball scooter,” like having bought a Russian car. He stated the difficulty to getting parts, and would rather not charge me the $60/hour just to find reliable parts sourcers online; he’d rather be paid to actually fix bikes.

He took it on me to find the parts myself, and supply them to him once I have. The most critical parts I need are brake rotors and brake-lines (He said cables, IIRC.) This is because the brake-lines were “tightened to their limits.” (I didn’t ask, “So how does one loosen them again?” But I hope you can answer that as question #3.) He also mentioned something about a hydraulic going out for the front. Who knew that brakes were hydraulically-operated on the front, but cable-operated on the rear? (<-- #4)

Now you see, my problem with finding parts suppliers, for this ‘’obscure’’ model, first off, would be by making sure they’re reputable, reliable and genuine. Google isn’t exactly all that great at telling how shady or legitimate these parts suppliers are.

Therefore,

1. What can you tell me about your experiences with United Motors motor-scooters?

2. What parts suppliers would you find trustworthy who supply for United Motors motor-scooters, and how did they earn your trust in the first place? (Caveat: This should go without saying, but I must order from them online.)

3. (See remark about loosening “tightened” brakes.)

4. (See pointed-at remark.)

I don’t care too much about cosmetic damage, as long as all important functions work right, and the Matrix 150 still gets me to my destinations on-time. Thanks kindly. --129.130.238.45 (talk) 16:39, 8 November 2012 (UTC)[reply]

There are limits to what people here can do - they can provide you with information and references about your scooter, but according to the guidelines (and common sense) they're not able to give you advice or to recommend suppliers. Any troll here could tell you to just bang on your brakes a few times with a left-handed monkey wrench and see if you stop better, and Wikipedia's disclaimer says they're not liable to you for a penny. Wnt (talk) 16:51, 8 November 2012 (UTC)[reply]

Process, Protuberance, Condyle

Hello. In anatomy, what is the difference between a process, a protuberance, and a condyle? Thanks in advance. --Mayfare (talk) 16:42, 8 November 2012 (UTC)[reply]

A protuberance is anything that protrudes, i.e., sticks out -- it is not really a technical term. A process is an extension that attaches to something else or connects to something else. A condyle is specifically a protuberance on the end of a bone. Looie496 (talk) 16:58, 8 November 2012 (UTC)[reply]

Is global warming necessarily bad for us?

I don't want to enter into the more basic questions like whether it's really happening or whether it's man made. I believe both to be true, although for me its existence is easier to prove than that man is at fault. Anyway, where is the evidence that it's necessarily a bad thing? And not neutral or positive at times? Or only marginally negative? OsmanRF34 (talk) 18:06, 8 November 2012 (UTC)[reply]

Well, it depends on how you define "us". For some places, there will likely be positive benefits, as agriculture will improve, for example, or places become more "livable". However, since most of the world lives within a few scant feet of sea level, most of the world is likely to see serious disruption to their lives. A city like Venice is already essentially unlivable, except that nostalgia keeps people from abandoning it entirely. Look what an extra 12 feet of water did to New York City. And that was just a storm surge, not a permanent situation. Imagine just raising 12 feet of water permanently. There's also the idea that extra heat in general leads to more energy within the weather/climate systems, leading to more dramatic swings between highs and lows of all sorts. Global warming doesn't just lead to universally warmer temperatures and nothing else: many models indicate that global warming leads to more variation in climate: rather than relatively uniform temperatures and rainfall from year to year, we could find years of extreme heat followed by extreme cold, years of drought followed by years of floods, more extreme at both ends. That level of unpredictability is hard to deal with. There's also the idea that increasing temperatures generally will increase storm activity: storms form because of rising warm air off of warm oceans; warmer earth means warmer oceans means more storms on average. Again, however, that doesn't mean that every single square inch of the earth will end up worse off. Some places which are now desolate wastelands without any agricultural productivity could turn into major farming belts, for example. --Jayron32 18:23, 8 November 2012 (UTC)[reply]
Also see The FAQ Q19. The speed of current climate change may seem slow to individual humans, but probably is unprecedented in nature. Ecosystems have problems adjusting at matching speed. Just imagine how long an oak forest needs to reach maturity, or how much smaller the ecosystem for marmots become if their preferred climate zone moves up a mountain. --Stephan Schulz (talk) 18:37, 8 November 2012 (UTC)[reply]
Canada, for example, should benefit. In addition to vast areas becoming farmable, there's the opening up of the northwest passage for shipping. And most of their major cities aren't near sea level (Toronto, Quebec City, Ottawa). There are a few, though, like Vancouver and parts of Montreal. StuRat (talk) 18:41, 8 November 2012 (UTC)[reply]
There was a report that came out recently that many of the northern countries - US and northern Europe - would suffer little if any economic damage, but the warmer parts of the world would be very severely affected. I'll see if I can run it down later. Wnt (talk) 19:04, 8 November 2012 (UTC)[reply]
I wouldn't include the US on that list. With major cities on the sea, like New York City, New Orleans, Miami, Pearl Harbor, Los Angeles, San Francisco, Seattle, Washington, DC, etc., and most of it subject to both hurricanes and tornadoes, it stands to lose quite a bit. Only Alaska will improve. StuRat (talk) 19:13, 8 November 2012 (UTC)[reply]
Stu, your comment that Canada "should benefit" is way off base. Climate change is likely to increase the incidence of droughts, floods, heat waves, and forest fires in Canada. That is bad for agriculture, not good. The IPCC is the authoritative body for scientific implications of global warming, and covers several of these topics. In fact, Canada has already suffered increased economic damages due to global warming, via the mountain pine beetle In fact [31], there is evidence that this process is a positive feedback to global warming. SemanticMantis (talk) 23:58, 8 November 2012 (UTC)[reply]
Unless global warming does at least 10,000 years worth of erosion to the Canadian shield and increases PPFD, warmer weather will not open up any land in Canada for agriculture. Also, the migration of typically southern pests north, and a faster maturation period for insect pests already here, will hurt Canadian agriculture. 50.101.137.171 (talk) 01:05, 9 November 2012 (UTC)[reply]
PPFD ? Are you saying the soil in northern Canada is incapable of sustaining crops ? StuRat (talk) 05:02, 9 November 2012 (UTC)[reply]
plus, you can't just take plants that evolved to grow in one latitude and grow them in a higher latitude, just because the temp is the same, since they tend to time their growth cycles, maturity, etc. by length of day (which is much less variable than temperature, and therefore less likely to trigger the plant into trying to go to seed in a warm May rather than the usual August, etc.). Onions, for instance, which come in long day and short day varieties. Anecdotal evidence: a friend of mine who attempted small-scale farming in northern Alberta, and claims to have grown vegetables which didn't have enough time to polymerize all the sugars into starch, cellulose, etc. and would break down into slush/dishwater at any attempt to cook them. Gzuckier (talk) 03:20, 9 November 2012 (UTC)[reply]
You should use crops which grow in warm places at the same latitude, then. Due to the Gulf Stream, Northern Europe is relatively warm, despite the northern latitude, so try those crops. StuRat (talk) 04:48, 9 November 2012 (UTC)[reply]
Good point, actually. Gzuckier (talk) 19:04, 9 November 2012 (UTC)[reply]
PPFD - photosynthetic photon flux density, or the related, Photosynthetically active radiation, here's a world map of PAR by month, [32] Even in July, light levels are too low in the northern half of Canada for plant agriculture. And yes, most of Canada's soil is not arable, class 7 land. [33] All the pink in that map will never be used for agriculture, and that's a map of Ontario. The last ice age scrapped away the soil, leaving only the rock underneath. 50.101.137.171 (talk) 16:31, 9 November 2012 (UTC)[reply]
One option would be to move soil there from once fertile lands to the south, which are now subject to drought. That soil map also seems to show that the soil is poor at mid-latitudes, but again improves at northern latitudes. (BTW, what were they thinking, using yellow to mean 3 different things, instead of using a different color for each ? Maybe they were originally different colors, but the pic was lowered to 16 colors when saved once.) I'd also like to see the soil quality map for the rest of Canada. StuRat (talk) 17:31, 9 November 2012 (UTC)[reply]
Umm, Canada is a little bit on the large size, you might need to take about all the soil in Europe to cover up the Canadian shield and tundra. The colours really are that poorly chosen, I've read the original books. For other provinces, see [34]. 50.101.137.171 (talk) 20:43, 9 November 2012 (UTC)[reply]
I wonder, since China has a building boom right now, are they being smart and avoiding building on the coast ? StuRat (talk) 19:15, 8 November 2012 (UTC)[reply]
oh dear, now i wish your question hadn't impelled me to search that out. "Nuclear construction set for coast by 2015" Gzuckier (talk) 03:26, 9 November 2012 (UTC)[reply]
And are they building it just like the Fukushima Daiichi Nuclear Power Plant ? StuRat (talk) 17:31, 9 November 2012 (UTC)[reply]
I'm guessing they're built entirely out of Chinese drywall. Gzuckier (talk) 19:25, 9 November 2012 (UTC)[reply]
Yes, there is evidence that global warming is "bad" (in terms of economic damages to human society, not to mention many other factors). For a brief overview, take a look at the latest IPCC report. SemanticMantis (talk) 00:07, 9 November 2012 (UTC)[reply]
There can also be sudden events that could cause our civilization to collapse. E.g. if due to droughts and floodings, agriculture fails for a few years in a row in China, India and Russia, then that will cause big problems, because the total food production then isn't sufficient to meet the demand. Countries will then have to close their markets to prevent everything from exported away to China, but then that would lead to the collapse of the World economy. If e.g. Saudi Arabia can't buy grain using their oil dollars, there is no point in selling any oil. Count Iblis (talk) 01:50, 9 November 2012 (UTC)[reply]
Those with enough money could still buy food, since higher food prices would make more expensive food production methods profitable, like greenhouses, hydroponics, etc. People with no food and no money would all starve. StuRat (talk) 04:54, 9 November 2012 (UTC)[reply]
The main thing you may think is that: "I am living up in the hills, I won't be impacted by raising sea levels". But one thing you can be absolutely sure, is that the coast-living people will flood towards you, to seek refuge, and beleive me, millions and millions of refuges arriving in your area will seriously change you life. On top of this, you need to take into account that there will much less farmland available, which means food prices will raise automatically.--Lgriot (talk) 09:33, 9 November 2012 (UTC)[reply]
  • I found that report I mentioned. Reading through this can be exasperating, but you can eventually get to a hundred pages about the methodology; it's not all colored circles and ideograms. The gist is that U.S. Russia, Europe, Australia, Argentina are OK, but Africa and Southeast Asia are screwed. Wnt (talk) 17:33, 9 November 2012 (UTC)[reply]
So long as we don't go nearly as far as Venus or Mars temperature wise, or have a climate like some planets where 700km/h wind storms are common... 50.101.137.171 (talk) 20:43, 9 November 2012 (UTC)[reply]

Perfect circles

This is a purely theoretical question, I understand perfect circles do not exist and probably never shall, but could someone answer this for me?

If perfect circles existed, as in Aristotle's wheel paradox, then a small circles circumference could be made to trace a ridiculous length by being attached to a larger circle. My question is this, would this make it possible to create infinite energy?

Different sized cogs turn at different rates due to the way their teeth are arranged. This is due to how many teeth fit on cogs relative to their size and the size of the cogs around them, right?

But if perfect circles existed, surely systems could be developed where cogs were replaced with completely circular pieces of metal or wood (as a circle could be described as a cog with an infinite amount of infinitesimal teeth), right? And if this was so different sized circular pieces, unlike cogs, could turn at the same rates regardless of their size. Going by set theory both would have an amount of points in their circumference and the teeth could be put into 1-1 correspondence.

Now, my question is this, given that all cogs/circles could turn at the same rate regardless, could a cog-based device be created that gave infinite energy? If so how? Equations would be greatly welcomed!

Note: If you disagree with the idea that the points on the cogs/circles would be able to be put into one-to-one correspondence, please just humor me and assume that all circles could turn at the same rate regardless to address the question!

(I have also posted this in the math desk as I was unsure if it was a math or physics question)

— Preceding unsigned comment added by 31.52.169.134 (talk) 19:24, 8 November 2012 (UTC)[reply]

A perfect circle is a mathematical abstraction. You can't build physical objects out of ideas. μηδείς (talk) 19:34, 8 November 2012 (UTC)[reply]

I realize this is not a real world question that is directly applicable to our surroundings but in a universe where all the laws of physics were exactly the same except perfect circles exist, would this be possible? — Preceding unsigned comment added by 109.157.115.249 (talk) 19:38, 8 November 2012 (UTC)[reply]

Regardless if you use circles or cogs, and regardless of whether they are perfect circles, a cog system does not create any energy. The only thing it does is transfer energy from one cog to the other. By using differently sized cogs/circles it is theoretically possible to create an infinite amount of torque, or to make an infinitesimally small circle spin at 'infinite' speeds (disregarding relativity for convenience), but the amount of energy put into the system is what you can get out of it, minus the energy lost due to friction. - Lindert (talk) 19:42, 8 November 2012 (UTC)[reply]

The premise of the question is flawed, in any event. Even if the center circle were perfect and contained an infinite number of cogs, they are still slipping. I assume you got this idea from the last sentence of the article you linked, but the infiniteness of the points on either wheel is irrelevant to the fallacy. Someguy1221 (talk) 19:52, 8 November 2012 (UTC)[reply]

I actually got it from this book:

http://www.amazon.co.uk/Brief-History-Infinity-Quest-Unthinkable/dp/1841196509/ref=sr_1_1?ie=UTF8&qid=1352404538&sr=8-1

It contains a much more detailed description of the paradox and highlights the flaws in all current theories and explanations to the paradox. That page does not give as much detail as this. Does anyone have an equation for energy transfer between cogs? — Preceding unsigned comment added by 109.157.115.249 (talk) 20:01, 8 November 2012 (UTC)[reply]

This is one of Zeno's paradoxes that he didn't get around to. The idea is that a cog of any size, turning, goes through an infinite number of positions, so they all have potentially the same number of teeth. However, the limit of the number of teeth with finer and finer cogs is always according to their sizes, and this mathematical abstraction remains irrelevant. The comical way to put it is - the cogs have an infinite number of teeth, which each mesh with their counterparts at a different moment -- therefore it takes an infinite amount of time for them to complete a revolution... :) Wnt (talk) 20:33, 8 November 2012 (UTC)[reply]

Didn't Zeno show that an arrow can go through an infinite amount of positions in a finite amount of time though? Surely this concept could be replicated? If you have a small cog and spin it, the speed isn't effected by the number of teeth is it? — Preceding unsigned comment added by 109.157.115.249 (talk) 21:37, 8 November 2012 (UTC)[reply]

Well, I was being somewhat humorous - the point of the paradox is that an infinite series can have a finite sum; that the duration of a moment can be infinitesimally small. Nonetheless, in relativity, space and time can be exchanged in different frames of reference. If infinity meant a one-to-one correspondence, then the infinite number of teeth in the cog would correspond to an equal infinite number of moments in the progression from past to future ... which are defined, I suppose, in terms of the period being considered. In other words, it's just absurd. Put it this way: infinity of any given type, multiplied by 5.2, equals what? Infinity. So the cogs have a truly undefined and undefinable gearing ratio when they have infinite numbers of teeth - there's no way to say how fast they turn or how fast relative to one another - it's all just a singularity in your equations. The way physics deals with that is to pick some other set of equations. Wnt (talk) 17:44, 9 November 2012 (UTC)[reply]
The OP did not ask for either an opinion or a prediction about a future event, so your hat comment is, at best, misleading. It's completely legitimate to ask about an ideal universe, because every single scientific law ever invented is only true in an ideal universe. Newton's laws, for example, are only approximations that aren't even good enough for everyday purposes (like GPS); do you plan on hatting every question that asks about Newton's laws, on the grounds that they don't apply to the real world?
First, I want to point out that ideal circles can exist in a Newtonian, special relativistic, or general relativistic world, but in the first 2 cases, conservation of energy is a result of something much more fundamental than different-sized circles: Noether's theorem. If any physical laws are symmetric with respect to time, then no matter what the laws actually are, conservation of energy holds. Energy is not conserved in general relativity, but an excellent approximation of it holds on small scales. Only when you get to cosmological scales where the Hubble expansion dominates is the conservation of energy obviously violated.
Second, I don't know whether the points could be put into 1-to-1 correspondence, but that is completely irrelevant. A transfer of energy is accomplished by work, and work is force applied over a finite distance. Right away that should tell you that you can't analyze points; you should be starting out by analyzing some finite distance, then take the limit as the distance goes to 0. --140.180.252.244 (talk) 05:04, 9 November 2012 (UTC)[reply]
Um no, scientific law are true in this world, as well as an ideal world. Newton's laws don't work for GPS as he didn't know about relativity (ie they are wrong for fast moving objects and high gravitational fields). Einstein corrected this error by coming up with general relativity which does allow GPS to work in this universe (which is not an ideal one). But Newton's laws do work at low speed and low gravitational fields (in this world as well as an ideal one). Thermodynamics work in this universe ( which is not an ideal universe). Laws of science work here, if they don't work here then they are wrong, or we don't have a full picture. Otherwise we wouldn't be able to predict anything. Dja1979 (talk) 15:20, 9 November 2012 (UTC)[reply]
This looks like it's related to the somewhat elastic size of countable infinity, in that there are an infinite number of points in the circumference of both circles, and you can make a 1:1 correspondence between them, but that doesn't mean they get to the end (i.e. one full rotation) at the same time, since you never reach the end of your count. Kind of like Hilbert's paradox of the Grand Hotel. Just off the top of my head. Gzuckier (talk) 19:22, 9 November 2012 (UTC)[reply]

Could dark matter be individual quarks?

To my knowledge most matter consists of arrangements of 3 quarks . The quarks are held together by the color force and have not yet been detected individually as the color force is so strong.

Dark matter particles do not interact with us except through gravitation so could dark matter simply be individual quarks that are not bound together via the color force?

The individual quarks could perhaps have been formed along with hydrogen in the early universe.

31.51.239.254 (talk) 20:16, 8 November 2012 (UTC) ap[reply]

Quarks have more than color force: they also all have electromagnetic charge, so unbound quarks would still interact with other matter even if they could be found in an unbound state. --Jayron32 20:23, 8 November 2012 (UTC)[reply]
Isn't by definition a quark s a fermion that interacts via the colour force. A fermion that doesn't interact via the colour force is called a lepton. Dja1979 (talk) 20:35, 8 November 2012 (UTC)[reply]
According to most mainstream models of quantum mechanics, quarks simply cannot exist individually -- so far, all attempts to find individual quarks have failed, and all attempts to create them artificially have only resulted in the creation of additional quark-antiquark pairs given off as mesons. 24.23.196.85 (talk) 23:06, 8 November 2012 (UTC)[reply]
At my local particle accelerator, they often invite expert scientists to give public lectures about dark matter. Here's the last one I attended, Deep Science: Mining for Dark Matter (full video available online - you might enjoy watching it). I recall walking away that evening somewhat with the dismal impression that most dark matter is just very low density, low temperature hydrogen whose blackbody temperature peak wavelength is very long and whose radiated intensity is very low. In other words, it's large quantities of boring, cold, non-interacting gas. It sounds a lot more boring when phrased that way! We can speculate ad infinitum about properties of matter we can't see, and speculate how it might interact with things it will never encounter; but at some point such speculation ceases to be scientific. Anyway, at least a distinction should be made between "dark matter" in the general sense, and rare exotic weakly-interacting particles (such as the WIMP, in all of the many speculative and potentially non-baryonic forms that it takes). One of the hard parts about observational cosmology is that you can't see things that are dark - so you've got to draw scientific conclusions from very rigorous theoretical study, and a lot of careful consideration of the null hypothesis. Nimur (talk) 23:41, 8 November 2012 (UTC)[reply]
The CMB shows that it isn't "large quantities of boring, cold, non-interacting gas" it is something else. Other evedence is bullet cluster which show that the centre of mass is differnt to the centre of light (hydrogen gas) of two super clusters of galaxies that have travelled through each other. This means the majority of the mass is made of something that very weakly or doesn't at all interact (except through gravity) with itself or other particles. The only discovered particle that fits this discription is the neutrino but the CMB data tells us that the ordinary (3 types of) neutrino is not the answer so must be something else not discovered yet. Dja1979 (talk) 01:12, 9 November 2012 (UTC)[reply]
I beg to differ. The cosmic microwave background doesn't show anything. The bullet cluster doesn't show anything. Cosmologists look at these things. The CMB is just an observed effect we see when we look into space, and as it is not alive, it is unable to perform scientific demonstrations for us. Scientists draw conclusions by observing the CMB, and other observable effects. You can agree with whichever scientific conclusion you believe rests most solidly on the observational evidence; but I consider it invalid to state that the evidence itself shows anything. Many scientists believe that these observations support theories of exotic matter. I am not convinced. Nimur (talk) 15:42, 9 November 2012 (UTC)[reply]
The bullet cluster does show something. The majority of the mass is in a place that is different from the majority of the stars. This can be explained by the majority of the mass being an unknown particle that doesn't interact with anything much, so passing through relativly unimpeded, and the gas and dust of the galaxies that passed through each other causing drag so seperating from the mass. How does mostly hydrogen do this? The CMB agains shows something, okay you can't interact with it but it holds information which a theory has to explain. If you don't have dark matter then the data doesn't fit the theory of the big bang with only ordanry matter. So if you don't believe that dark matter causes the difference, then you have to come up with another way of explaning the data (the oscillations in the power spectrum of the CMB). Dja1979 (talk) 17:41, 9 November 2012 (UTC)[reply]
You place a higher degree of confidence in the evidence of gravitational lensing than I do. And you are not alone: a very large number of very prominent scientists agree with you - including the lecturer I linked above. In fact, NASA even published the phrase "direct proof" of dark matter in one of their Bullet Cluster press-release, attributed to a researcher affiliated with the Chandra program. However strong these words are, and despite my immense respect for many of these researchers, I am still not convinced. Nimur (talk) 20:21, 9 November 2012 (UTC)[reply]
When someone disagrees with the overwhelming consensus of the experts in the field, and said person is not an expert himself, it's his responsibility to demonstrate why the experts are wrong. It's not the experts' responsibility to disprove his theories. Otherwise, science can't progress, because there's always someone who will invent theories with no support and say "prove me wrong". --140.180.252.244 (talk) 22:38, 9 November 2012 (UTC)[reply]
Nimur, please don't speculate about things you know nothing about. It's absolutely not true that any substantial portion of dark matter could be hydrogen gas. In fact, there's very little doubt that dark matter is made up of WIMPs--weakly interacting massive particles. I'll try to summarize the evidence, but basically Dja1979 hit the main points.
  • A spectral analysis of the cosmic microwave background gives numerous peaks. The second peak is determined by the baryon-photon ratio; all ordinary matter, including hydrogen, is made up of baryons. The photon density of the universe can be accurately calculated because most of the universe's photons are from the CMB, and the CMB is modelled very well by a blackbody. From these data, the baryon density of the universe can be calculated, and it turns out to be about 4% of the universe's total energy density. We know from independent measurements that matter accounts for 27%, so almost 90% of the universe's matter cannot be baryonic, and certainly can't be hydrogen.
  • Anisotropies in the CMB were predicted to be on the order of 1e-3. It turns out that in actuality, they were on the order of 1e-5. If only normal matter existed, the CMB anisotropies would not have been large enough to form the structure we see today. There must have been a large number of particles that decoupled from the photon-matter fluid long before baryonic matter, and provided the seeds for the large-scale structures seen today. The very fact that they decoupled before baryonic matter means they're weakly interacting.
  • Dark matter distributions have been mapped out using various techniques, including gravitational lensing, the virial theorem, and measurements of galaxy rotation curves. Dark matter is observed to lie in spherical halos around galaxies, and the Bullet Cluster shows that these dark matter halos have a behavior distinct from the rest of the galaxy (i.e. they don't interact as much). Hydrogen doesn't form spherical halos like this; it tends to coalesce, heat up dramatically, and form star clusters at the slightest perturbation. The fact that spherical halos exist mean that the particles in them must be weakly interacting--both with photons and with themselves--and massive, because if they had low mass, they would be moving too quickly to coalesce into halos.
  • Don't imagine that astronomers can't observe dark, cold hydrogen gas if it filled the universe. Even if it doesn't emit light, absorption lines would be painfully obvious at every redshift, in the spectrum of every extragalactic object. As it turns out, astronomers do observe numerous Lyman alpha forests in extragalactic spectra, but they're not at all close to the amount needed to explain dark matter.
  • The only N-body simulations that reasonably simulate the structure we see today, starting from the initial conditions imposed by the CMB, assume a large amount of cold (aka massive), weakly interacting matter. --140.180.252.244 (talk) 06:04, 9 November 2012 (UTC)[reply]
While you're making these good replies - my understanding is that supersymmetry took some hits lately. But is the old fashioned mirror matter still in contention? Could there actually be planets and stars that work like our own, but are invisible and non-interacting to us, except by some very weak coupling of mirror photons with us? Wnt (talk) 09:47, 9 November 2012 (UTC)[reply]
I think most people expect dark matter to be the neutralino as this is the lightest supersymetric particle in most supersymetric models. As you say certain model of supersymmetry have been disproven but there's alway a version to take it's place we have a long way to go before we disprove all of supersymmetry. As for mirror matter it seems unlikely. I refere back to the bullet cluster example. There most of the mass of the galaxies didn't interact with each other when they passed through each other. If dark matter was like ordinary matter with dark light dark quarks etc then they would of interacted with each other so producing drag. So it seems that what ever dark matter is, it interacts with itself as much as it does ordonary matter. Dja1979 (talk) 15:05, 9 November 2012 (UTC)[reply]
I find it alarming that I have, thus far, provided the only external reference to a web-resource on the cosmology of dark matter - a video presentation at the facility where the quark was discovered, presented by an expert cosmologist, available free on the internet. If we have any more highly-trained, world-renowned expert cosmologist contributors who know more about this subject - or even if there are any well-informed non-expert enthusiasts present - why don't you provide some links to texts, papers, books, videos, or web-resources? I'd love to inform myself more on the topic. In the spirit of full disclosure, I am not currently a professionally-employed-cosmologist, but I think it's a little unfair to say I know nothing of the subject. Nimur (talk) 15:48, 9 November 2012 (UTC)[reply]
I should clarify that when I said "you're making good replies" above, I was speaking plural, including you, Nimur, who are absolutely of the very best contributors on this Refdesk overall. But I also appreciate a good fight over these ideas to make clearer where the action is. :) Wnt (talk) 17:06, 9 November 2012 (UTC)[reply]
I watched that video. Not only does Golwala not say dark matter can be hydrogen, he says exactly the opposite. In fact, he recounts almost every point I've made for why it cannot be hydrogen. I'll paraphrase a few quotes, because they're too long to copy exactly:
14:00: "The red here shows X-ray emission from normal matter. You see that the normal matter is not lined up with total matter, which itself is interesting.
15:30: "You've got 1 component that collides...it interacts like normal matter does...but then you've got a large chunk that doesn't interact."
16:15: "You need something that's not the normal matter we know about to explain how this thing is behaving."
16:30: "Can it be normal matter? Well, for one thing the dark matter has to be collisionless, and you can't have that with normal matter. There are cosmological measurements that strongly indicate normal matter is only 1/6 the total amount of matter..there's the abundances of light elements, we understand very well how they were produced, and can use the measured abundances to determine how much normal matter there is in the universe. And it says normal matter only contains 1/6 the total mass we measure in numerous ways. There's also the CMB...fluctuations are seeds of all the structures we see today...the fluctuations are actually too large for them to be caused by normal matter, because at the time, normal matter couldn't collapse. In some sense, the fluctuations were also too small...they're a factor of 10 too small to cause the structures we see today."
19:00: "Let's hypothesize a new particle."
21:00: possible explanations with modified gravity
22:50: "Modified gravity doesn't seem to fit the data as well as dark matter"
24:20: "This particle must interact with gravity and the weak force, and no others, because if it did, we would have already seen it."
25:25: "They have to have a mass of about 100 proton masses, give or take a factor of 10, and have to move at about the same speed as other stuff in our galaxy."
The rest of the video talks about various experiments to detect WIMPs. To answer Wnt, the mirror particle idea seems unlikely for exactly the reason Dja1979 gave: dark matter particles have to interact weakly with themselves, not just with normal matter. Specific candidates for dark matter are much more speculative than the fact that dark matter exists, and that it cannot be baryonic. Don't confuse the claim "dark matter is something we've never seen before" with "dark matter is particle X, where X=neutralino (or whatever else)" If somebody with green skin and 3 arms started robbing you, you can be very certain that you've never seen this person before, but identifying the person is much harder. --140.180.252.244 (talk) 17:57, 9 November 2012 (UTC)[reply]


Mirror matter could be part of the dark matter. It is possible that dark matter isn't all neutralinos, axions, sterile neutrinos etc. but a combination of all of these candidates. The constraint from the Bullet cluster does leave room for some mirror matter models, because it implies a constraint on the cross section that depends on the density which is easy to evade. Count Iblis (talk) 17:05, 9 November 2012 (UTC)[reply]

November 9

Antimatter meteor/comet

What would happen if an antimatter meteor or comet hit the Earth? --168.7.238.36 (talk) 06:22, 9 November 2012 (UTC)[reply]

A large one would pretty much destroy the planet, while a small one might just destroy all life on Earth. However, if it hit at a shallow angle, the initial explosions when it first contacted the atmosphere might blast it back into space before it did much damage. Fortunately, there don't actually seem to be any. Within our solar system, I suspect that even the sparse gas here would cause sufficient explosions on it's surface to break it up. StuRat (talk) 07:27, 9 November 2012 (UTC)[reply]
Incidentally, are you aware that one theory is that the Tunguska event was a tiny antimatter meteor which struck Russia ? StuRat (talk) 17:42, 9 November 2012 (UTC)[reply]
See antimatter comet. StuRat (talk) 17:44, 9 November 2012 (UTC)[reply]

Lead poisoning

I have an extension PCB(a pci card of some sort) that is not compliant with ROHS That is built with lead-solder, DOES handling it pose a threat to health or something of that nature, like getting poisoned if you are to have contact with the solder joints on it? Should you always wash your hands thoroughly after handling it? — Preceding unsigned comment added by 77.35.32.100 (talk) 09:38, 9 November 2012 (UTC)[reply]

There is nothing to be concerned about. No need to wash hands beyond the usual requirements. Electric and electronic devices have been made with tin-lead solder for around 100 years without any problems whatsoever from handling. If there was any problem, it would have occured with the factory people doing the soldering - and for the first 70 years or so it was mostly done by hand. No problems have been reported. Same with electronics hobbyist and radio hams - who have always, and still do, assembled their devices with tin-lead solder. Problems are unknown. I have an electronics background myself. Employers generally banned the consumption of food at the workbench, and recommended washing hands before eating. But this was more about keeping the workplace clean and avoiding the transmision of microbes (thereby keeping teh workers healthy and not off sick) than about lead. Later I worked in a diesel engine facility, and they had the exact same rules, and zero use of lead.
In circuit boards of at least reasonable quality, after soldering, the solder side of the board is coated with a type of varnish. This varnish is very thin and generally is not visible.
You need to understand the reason for the use of lead free solder in recent years. It has been determined that very small amounts of lead affect intelligence, and most people have medically significant lead in their bodies. First, various countries banned lead in gasoline. That had little effect, except in the USA, where the number of gasoline engined cars and trucks per unit area is much greater than in other countries (and even in the USA, significant lead levels in humans still occur). So the Europeans decided to ban the use of lead altogether (an exemption has been made for lead-acid batteries). However, lead is still ubiqitous in the environment. One of the major causes of lead in our bodies is the use of lead-based paint in houses. In most advanced countries it was banned decades ago, but every house gets sanded back and repainted once in a while, releasing the lead as dust into the air. There are lots of other reasons for lead in the environment that far swamp out any nanoscopic source in electronic cards.
Keit 120.145.147.142 (talk) 10:35, 9 November 2012 (UTC)[reply]
I should add that one problem with lead in landfills, or any stable element, is that it doesn't decay. So, while something like feces ceases to be a problem after it's been buried a few years, this isn't the same with lead. It's permanent. As new layers of soil are deposited on top, the lead will eventually be buried so deeply that we will be unlikely to have much contact with it, but this will take thousands of years. StuRat (talk) 17:24, 9 November 2012 (UTC)[reply]
The lead in the solder would be released back into the environment if it is discarded into the earth and the lead gradually corrodes over the years. As others have mentioned there is no hazard by handling it. But don't eat it or grind it to dust. Graeme Bartlett (talk) 11:01, 9 November 2012 (UTC)[reply]
Yeah, like everybody says, the major health risks of lead-based solder are water pipes, where it's been banned for a long time, and for the solderers who are inhaling the vapors over a lifetime. Although, I would definiely recommend against eating a lot of it. Gzuckier (talk) 19:11, 9 November 2012 (UTC)[reply]

What was the purpose of the Galileo thermoscope

I try to understand what were the purposes of Galileo galilei when he made the thermoscope? It seems that the him purpose was not been to make a scale of the heat, but he meant to see or prove something... Do you know what was it? מוטיבציה (talk) 14:00, 9 November 2012 (UTC)[reply]

Why do you discount the idea of making a scale of the heat? This was a non-trivial scientific and instrument-based question in the 17th century, and there was considerable debate at the time what temperature really was (e.g. was it a substance or was it something else? We now know that it is identical with movement on a very small scale, but they didn't know that then). --Mr.98 (talk) 14:29, 9 November 2012 (UTC)[reply]

Converting candela per square metre to "photon density"

Greetings!

Given the luminance (in candela per square metre) and wavelength of a light source, I'd like to obtain its "photon density" (a measure that's new to me but used to describe stimuli in some vision research papers; its units are given as photons/cm2/s). One paper specifies that a red light source has a wavelength of 635 nm, a luminance of 34 cd/m2, and a photon density of 1×1014 photons/cm2/s (I include these values as they can - presumably - be used to check that a computation using the first two values correctly results in the third).

The candela is defined as a measure of "luminous flux per unit solid angle"; if it could be converted to an energy measure, the conversion should simply be a matter of computing the energy per photon of the light source using , dividing the overall energy by the per-photon energy, and then accounting for the change from m2 to cm2. Based on the fact that 1 candela = 1 lumen⋅steradian and 1 lumen = 1/683 Watts, 1 candela should be 1/683 Watt⋅steradian. There are 4π steradians in a sphere, so for a light source emitting light equally in all directions 1 candela should be equivalent to 0.0184 Watts.

Coming back to the values given in the aforementioned paper, 34 cd/m2 becomes 0.63 W/m2 and the energy per photon for a 635 nm wavelength light source is 3.13×10-19 W⋅s (1 Joule = 1 Watt⋅second). Dividing the first value by the second (to obtain the number of photons) results in 2×1018 photons/m2/s ("photons" is a unitless quantity here in the same way that "cycles" is for frequency (cycles/s), so the units are technically s/m2); finally, dividing by 1002 to convert from s/m2 to s/cm2 results in 2×1014 photons/cm2/s.

As this result doesn't match the value given in the paper, something is evidently amiss. The only thing I can see that might be incorrect is the assumption that the source emits light equally in all directions, but I would appreciate any suggestions or corrections. Hiram J. Hackenbacker (talk) 18:05, 9 November 2012 (UTC)[reply]

If I'm reading it right, the basic problem I see is that the candela is a measure of light emitted from a source, while the the photon density is a measure of the light received at the target. So, in astronomy terms, one is absolute magnitude and the other is apparent magnitude. The apparent magnitude varies not only with the absolute magnitude, but also the intervening distance and particles. So, there is no direct conversion between the two, and a conversion can only be done when you know what lies in between. StuRat (talk) 19:10, 9 November 2012 (UTC)[reply]
The candela is indeed a measure of the intensity of the source (and not directly comparable to luminance measures), but the stimulus in the paper is characterized by its luminance given as candela per square metre, which should be comparable to other per-unit-area measures (the paper used a Ganzfeld-type stimulus, which is basically a surface illuminated such that it has uniform luminance, which is why the surface is characterized by a luminance measure rather than one of luminous intensity). I converted candela to Watts via an assumption about the effective point source illuminating the Ganzfeld surface (which may or may not be valid) but this was just for the purpose of converting the given candela per square metre value to one expressed in more useful units, so the luminance of the surface is still a per-unit-area measure after the conversion and should remain comparable (and convertible, hopefully) to photon density. Hiram J. Hackenbacker (talk) 19:34, 9 November 2012 (UTC)[reply]


You may enjoy reading Labsphere's introductory guide to integrating photometry. This guide is not overtly commercialized: it actually provides a lot of solid physics and mathematics that is useful for people who are interested in quantitative measurement of light. A large quantity of educational literature is available on the company's website in the science section.
Particularly note their list of various units (page 2); it provides a handy breakdown between flux, flux-per-area, flux-per-solid-angle, and flux-per-area-solid-angle. Each of these quantities represents a different useful parameter in photometry and radiometry. The candela is a measure of photometric photons per solid angle - approximating a standardized human perception of light - so you can't directly convert it to total number of photons. This critical distinction is elaborated in the guide, and is covered in our article on spectrophotometry. Nimur (talk) 20:00, 9 November 2012 (UTC)[reply]

Deer

Why do mother deer run off their young bucks?--Wrk678 (talk) 23:12, 9 November 2012 (UTC)[reply]